Anda di halaman 1dari 75

Dosier mdulo IIII

Mdulo III

1. Introduccin
2. Razones y frmulas trigonomtricas
2.1. ngulos
2.1.1. Medicin de un ngulo
2.1.2. Terminologa
2.2. Razones trigonomtricas para ngulos agudos
2.3. El crculo unitario
2.4. Razones trigonomtricas para ngulos destacados
2.5. Iden dades Trigonomtricas
2.5.1. Las iden dades bsicas
2.5.2. Iden dades de suma y resta de ngulos
2.5.3. Iden dades de duplicacin y linearizacin

7
7

2.6. Tabla resumen de iden dades trigonomtricas ms importantes


2.7. Lecturas y ejercicios complementarios
2.8. Problemas
3. Resolucin de tringulos y ecuaciones trigonomtricas
3.1. Ecuaciones trigonomtricas
3.1.1. Ecuaciones de la forma A cos (x ) + B sen (x ) = C
3.2. Lecturas y ejercicios complementarios

21

4. Transformaciones en el plano: isometras, traslaciones y rotaciones


4.1. Isometras: Simetr as
4.1.1. Simetr as Centrales
4.1.2. Propiedades de las simetr as centrales
4.1.3. Simetr as Ortogonales
4.1.4. Propiedades de las simetr as ortogonales
4.1.5. Algunas ac vidades para realizar en el aula
4.2. Traslaciones
4.2.1. Ac vidad
4.2.2. Propiedades de las Traslaciones
4.3. Rotaciones
4.3.1. Rotacin en el plano orientado
4.3.2. Ac vidad
4.3.3. Propiedades Esenciales

27

4.4. Transformaciones en el plano: composicin de simetras y homotecias


4.5. Ac vidad 1
4.6. Ac vidad 2
4.7. Ac vidad 3
4.8. La isometra directa ms simple
4.9. Descomposicin de traslaciones y rotaciones como composicin de simetras axiales
4.10. Problemas de Traslaciones
4.11. Problemas de Simetras Centrales o Reexiones Puntuales
4.12. Problemas de Rotaciones
5. Transformaciones en el plano: homotecias
5.1. Teoremas Fundamentales
5.2. Homotecia de guras geomtricas fundamentales
5.3. Problemas de Homotecias
5.4. Composicin de Homotecias

54

En este mdulo se estudian los aspectos espaciales del mundo fsico que pueden ser tratados con el
lenguaje y los conceptos de la trigonometra y las transformaciones geomtricas en el plano. A partir de los
conceptos bsicos geomtricos y algebraicos se abordan los temas bsicos de la trigonometra en el plano,
a saber: el estudio de la relacin de semejanza entre guras geomtricas, el clculo de medidas de ngulos
y distancias, las razones e identidades trigonomtricas, la resolucin de tringulos y la construccin y
visualizacin de las funciones trigonomtricas y sus propiedades. Se hace nfasis en la visualizacin de las
guras y los aspectos geomtricos subyacentes y, siguiendo el enfoque del curso de geometra, se contina
haciendo un uso gradual del mtodo deductivo para justicar y formalizar las propiedades y relaciones
bsicas de las identidades y funciones trigonomtricas. Y con los movimientos o transformaciones
geomtricas en el plano se estudia el efecto geomtrico de una traslacin, una reexin, un giro, una
homotecia o una combinacin de ellas sobre una gura o conguracin geomtrica.

Razones y frmulas trigonomtricas

La trigonometra se dene como el estudio de las relaciones entre los lados y los ngulos de un tringulo
cualquiera. Estas relaciones se resumen en las llamadas razones trigonomtricas.
En el desarrollo del presente mdulo se asume que el estudiante est familiarizado con el trabajo con
tringulos (clasicacin, terminologa, rectas notables, ngulos dentro de un tringulo, rea, permetro,
teorema de Pitgoras) y que ene nociones bsicas de trigonometra en tringulos rectngulos.

2.1.

ngulos

Imagnese una lnea recta, comenzando de una posicin ja OA (Fig. 1a) y rotando alrededor de un punto
O en la direccin opuesta al movimiento de las agujas del reloj hasta llegar a la posicin indicada por OB .
Al rotar de OA a OB , se describe el ngulo AOB . De esta manera, tenemos el concepto de un ngulo
como formado por la rotacin de un segmento de recta alrededor de un punto jo, que llamaremos el
vr ce del ngulo. Como se puede observar en la gura 1a, si se toma cualquier punto C del segmento de
recta OA, este punto marcar con su movimiento el arco CD de un crculo. No existe lmite en la rotacin
de OA y, por consiguiente, se pueden formar ngulos de cualquier magnitud al rotar un segmento de
recta de esta manera. Suponga, por ejemplo, que la rotacin de OA se con na hasta la posicin OA0
(Fig. 1b), en los que A0 , O y A son colineales. El punto C habr delineado un semicrculo y el ngulo A0 OA
se le llama ngulo llano. Si dejamos que OA con nue su rotacin hasta que llegue a su posicin original,
habr realizado una rotacin completa y el punto C habr marcado la circunferencia de un crculo.
2.1.1.

Medicin de un ngulo

Por convencin, los ngulos se miden en sen do an horario. Cualquier ngulo formado a par r de la
rotacin de un segmento sobre un punto en el sen do de las agujas del reloj tendr una medicin nega va. El concepto de la formacin de un ngulo por rotacin nos lleva a un mtodo conveniente de
medir ngulos. Si imaginamos que la rotacin completa se divide en 360 divisiones iguales, tendremos
360 pequeos ngulos iguales, cada uno de los cuales ser llamado un grado1 . Note que sta medicin
1

Puede que se est preguntando el porqu el 360 como el nmero de divisiones de una rotacin completa para obtener
un grado. La seleccin de este nmero fue hecha de forma muy temprana en la historia de la civilizacin humana y sabemos,
por ejemplo, a par r de las inscripciones en templos que el nmero fue empleado por los an guos Babilonios. El nmero
B
D

A0

(a)

(b) ngulo Llano

Figura 1
8

II

A0

A0

O
IV

III

(a) Rotacin completa

(b) ngulo recto

Figura 2
de ngulos, aunque comnmente u lizada, no es la nica existente. Los gradianes, por ejemplo, dividen
al crculo en 400 partes iguales. Esta medicin fue adoptada en Francia para que el sistema de medida
de ngulos fuese consistente con otros sistemas de medicin mtricos.
Un tercer mtodo para medir ngulos es absoluto, en el sen do que
B
no depende de una divisin arbitraria del crculo formado por una revolucin completa. Para obtener la medicin, tmese un crculo con
centro O (ver gura 3). Rote el radio OA hasta la posicin OB , tal que
la longitud del arco AB es igual a la del radio. A la medida del ngulo
= 1 rad
A
AOB le llamaremos un radin. Note que la descripcin anterior nos
O
dice que la medicin en radianes de un ngulo est denida como la
razn de la longitud del arco denido por la rotacin que describe al
ngulo y la longitud del radio del crculo. Dado que la cirfunferencia
se ob ene con 2r , donde r es el radio del crculo, una revolucin
completa equivale a un ngulo de 2 radianes.
Figura 3
2.1.2.

Terminologa

La gura 2b representa una rotacin completa, como la mostrada en


la gura 2a. Sea E y F los puntos medios de los arcos entre C y D en cada semicrculo. Si trazamos el segmento de recta EF , este pasa a travs de O y la circunferencia queda dividida en cuatro partes iguales.
Cada uno de los ngulos COE , EOD, DOF y F OC representa a un cuarto de la rotacin completa y son
llamados ngulos rectos, conteniendo cada uno 90 grados. El crculo, adems es dividido en cuatro partes iguales llamadas cuadrantes que se numeran en el orden de formacin. Cualquier ngulo menor que
90 grados es llamado agudo y cualquiera que se encuentre entre 90 y 180 grados es llamado obtuso. Dos
ngulos cuyas medidas suman 180 y 90 grados son llamados ngulos suplementarios y complementarios,
respec vamente. Dos ngulos que enen la misma medida son llamados congruentes.
probablemente surgi de la divisin de los cielos por astrnomos an guos en 360 partes, correspondientes a lo que se tomaba
como el nmero de dias en el ao.

2.2.

Razones trigonomtricas para ngulos agudos




Suponga que  un ngulo agudo, es decir, 0 <  < 90 0 <  < . Con este ngulo agudo, se puede
2
formar un tringulo rectngulo con hipotenusa de longitud c y catetos de longitudes a y b, tal como se
muestra en la gura 4.

Figura 4: Razones trigonomtricas


Con estos tres lados se pueden formar la seis razones siguientes:

b
;
c

a
;
c

b
;
a

c
;
b

c
;
a

a
b

Se ene que estas seis razones dependen slo del ngulo  y no del tringulo rectngulo formado. En
efecto, si se forma otro tringulo con hipotenusa de longitud c 0 y catetos de longitudes a0 y b0 , entonces
los dos tringulos son semejantes y, por tanto, las razones correspondientes son iguales (ver gura 5).

Figura 5: Razones trigonomtricas


Como resultado se ob ene:

b b0
= ;
c c0

b b0
c c0
c c0
a a0
= ;
= ;
= ;
=
a a0
b b0
a a0
b b0
Como las razones dependen slo del ngulo  y no del tringulo rectngulo formado, se da a cada razn
un nombre especial que involucra al ngulo :
b
sin  = ;
c

a a0
= ;
c c0

a
cos  = ;
c

b
tan  = ;
a

c
csc  = ;
b
10

c
sec  = ;
a

cot  =

a
b

2.3.

El crculo unitario

Considere el plano cartesiano xy . Sea un crculo cuyo centro est en origen O del plano cartesiano y
ene radio igual a 1. Este crculo es conocido como El Crculo Unitario y la ecuacin de la circunferencia
es x 2 + y 2 = 1.
Sea P un punto sobre la circunferencia. Las proyecciones de P sobre el eje x y el eje y son respec vamente los puntos A y B . Por P se traza una tangente a la circunferencia, que corta al eje x y al eje y en
los puntos C y D, respec vamente. Finalmente, sea el ngulo orientado que se forma desde el eje x
posi vo hasta OP .

1. Si se supone que es un ngulo entre 0 y 90 , entonces de la denicin de las razones trigonomtricas se ob ene que AP = cos , BP = sen , CP = tan , DP = cot , OC = sec ,
OD = csc .
2. Si ahora suponemos que P es un punto cualquiera de la circunferencia y es un ngulo que toma
cualquier valor entre 0 y 360 , entonces se pueden generalizar las razones trigonomtricas de
la siguiente forma: los puntos de la gura enen por coordenadas P (cos ; sen ), A(cos ; 0),
B (0; sen ), C (sec ; 0), D(0; csc ); as
La funcin seno de viene dada por la coordenada en y del punto P .
La funcin coseno de viene dada por la coordenada en x del punto P .
sen
La funcin tangente de se dene como el cociente cos
, y su magnitud coincide con la
longitud del segmento tangente a la circunferencia P C .
, y su magnitud coincide con la
La funcin cotangente de se dene como el cociente cos
sen
longitud del segmento tangente a la circunferencia P D.
11

La funcin secante de viene dada por la coordenada en x del punto C , y su magnitud


coincide con la longitud del segmento secante a la circunferencia CO.
La funcin cosecante de viene dada por la coordenada en y del punto D, y su magnitud
coincide con la longitud del segmento secante a la circunferencia DO.
A par r de estas deniciones, se puede analizar el comportamiento de las funciones trigonomtricas cuando vara de 0 a 360 , o equivalentemente en radianes de 0 a 2 . Para ello complete
la tabla siguiente (coloque ND si la funcin no est denida):
0
sen
cos
tan
cot
sec
csc

0;

)

2
Crece de 0 a 1


2
1

(

;

2
Decrece de 1 a 0


0

)
3
; 2
Decrece de 0 a 1

3
2
1

)
3
; 2
2
Crece de 1 a 0

3. Observe que todas las funciones trigonomtricas f ( ) cumplen f ( ) = f ( + 2 ), por lo que


son funciones peridicas. Determine el perido de cada una de ellas.
4. Observe que para todo :
a)
b)
c)
d)
e)
f)

1  sen  1.
1  cos  1.
1 < tan < 1.
1 < cot < 1.
1  sec , o bien sec  1.
1  csc , o bien csc  1.

5. Aplicando el Teorema de Pitgoras a los tringulos 4OAP , 4OCP , 4ODP , se ob ene que para
todo se cumple:2
a) sen2 + cos2 = 1.
b) 1 + tan2 = sec2 .
c) 1 + cot2 = csc2 .

2.4.

Razones trigonomtricas para ngulos destacados

La siguiente tabla resume algunos valores del seno, coseno y tangente de ngulos agudos calculados a
par r de guras geomtricas conocidas:
2

Siempre que permita que la funcin trigonomtrica que se est evaluando est bien denida.

12

HH0 10

 
6


4

1
h

H
(b) Para 3 y 6

(a) Para 4

Figura 6: Clculo de seno y coseno para ngulos bsicos

cos

sen

tan


p6
3
2
1
p2
3
3


p4
2
p2
2
2
1


3
1
p2
3
2


2


2

0
1
ND


3


4


6

Es importante saber cmo se ob enen los valores de estos ngulos. Para 0 y 2 , los valores son obtenidos
directamente de los valores de las coordenadas x e y de los puntos sobre la circunferencia que los describen. Para 4 (g. 6a), notamos que el tringulo rectngulo es issceles (ambos
p2 ngulos agudos deben
1

p
sumar 2 ) y por el teorema de pitgoras, a = 2 , equivalentemente, a = 2 . Ya que a corresponde a
las coordenadas x e y del punto P (asumiendo que O se encuentra en el origen), tambin corresponde
al seno y coseno de 4 .Para encontrar el seno y coseno de 3 y 6 notamos que los ngulos son complementarios y que el tringulo rectngulo que los con ene es la mitad de un tringulo equiltero (ver g.
6b). Por geometra sabemos que la altura en un tringulo equiltero corresponde tambin a la bisectriz
y mediatriz. Por lo tanto, ya que OP = 1, sabemos que OH
= 12 (por ser P H mediatriz),p de donde el
p
cos 3 = sen 6 = 12 . Luego, por pitgoras, la altura P H = 23 , de donde sen 3 = cos 6 = 23 .

2.5.

Iden dades Trigonomtricas

Una Iden dad es una igualdad que es vlida para todos los valores que pueda tomar la variable (o las
variables, si hay ms de una).3 En Trigonometra hay muchas iden dades y en esta seccin obtendremos
algunas de las ms conocidas.
2.5.1.

Las iden dades bsicas

En cambio, una Ecuacin es una igualdad que slo es vlida para ciertos valores de la variable, llamados Soluciones de
la ecuacin, y la variable toma el nombre de incgnita.

13

Para obtener iden dades trigonomtricas a par r de nuestra


denicin de seno y coseno de un ngulo dentro del crculo
unitario, primero observamos que los puntos de dicho crculo asociados a ,  ,  + y son los vr ces de
un rectngulo, suponiendo que los ejes de coordenadas son
los ejes de simetra, como se muestra en la gura 7. De esta
observacin, podemos hacer una lectura directa de la gura
para obtener las siguientes iden dades:

cos( ) = cos
cos( ) = cos
cos( + ) = cos

sen( ) = sen
sen( ) = sen
sen( + ) = sen

Si ahora se realiza la construccin para obtener ngulos complementarios dentro de la circunferencia, como se muestra
en la gura 8, por congruencia de tringulos obtenemos las
siguientes iden dades:

cos
sen
2.5.2.

(

2
(
2

= sen

cos

= cos

sen

(

2
(
2

+ =
)

sin

cos

+



2

Figura 7

 +
2

sin

cos

sen

+ = cos

Figura 8

Iden dades de suma y resta de ngulos

Las siguientes iden dades a obtener sern aquellas que nos


permitan obtener el valor del seno y coseno de la suma +
y de la diferencia en funcin del seno y coseno de y
de . Encontraremos primero una iden dad para sen( + ).
Sea P el punto sobre el crculo unitario que describe al ngulo
a par r del eje de coordenadas x , y Q el que describe al ngulo
a par r del punto P , como se muestra en la gura 9. Sea C el
punto donde cae la perpendicular a OP que pasa por Q. Sean
B , D y A los puntos en OI donde caen las perpendiculares a OI
que pasan por los puntos Q, C y P , respec vamente. Finalmente,
sea EC paralela a OI . Los siguiente datos pueden ser extraidos
de esta construccin:

P
C

E

O

Figura 9

OQ = 1.
QC = sen y OC = cos .
Ya que DCEB es un palalelogramo, EB = CD.
CD
sen = CD
OC = cos , de donde CD = sen cos = EB .
14

Note que para obtener el valor del sen( + ), es necesario conocer el valor de QB . De los datos
anteriores obtuvimos el valor de EB , por lo que resta nicamente el valor de QE . Para esto, note que
por ngulos alternos internos entre paralelas \ECO = . Pero \QCE + \ECO = 2 y tambin
\EQC + \QCE = 2 , de donde \CQE = \ECO = . Sabiendo esto, tenemos que cos = QE
QC =
QE , de donde QE = cos sen . As,
sen

sen( + ) = QB = EB + QE = sen cos + cos sen


A par r de esta iden dad pueden deducirse las mencionadas en el siguiente teorema, cuyas demostraciones se dejan al lector en forma de ejercicios.
Teorema 2.1. Para cualquier pareja de ngulos y se cumple que

sen( + )
sen( )
cos( + )
cos( )

= sen cos + cos sen


= sen cos cos sen
= cos cos sen sen
= cos cos + sen sen

Demostracin.
a) sen( ) = sen cos cos sen
Deduciremos el resultado u lizando reas. Veamos la siguiente gura:
C

a
b

A

D
asen

Tenemos que: (ACD ) = (ACB )

acos

(ADB ). Calculado el rea de cada tringulo:


a(b sen( ))
2
(b cos )(a sen )
(ACB ) =
2
(a cos )(b sen )
(ADB ) =
2

(ACD) =

Sus tuyendo en la primera igualdad obtenemos el resultado deseado:

sen(

) = sen cos
15

cos sen

D
b

bsen


bcos

b) cos( ) = cos cos + sen sen


Procederemos como en el literal anterior u lizando reas.


2

bcos

asen

acos

A
bsen

En la gura tenemos que: (BCA) = (BCD) + (DCA), calculado el rea de cada tringulo:

(BCA) =

a(b sen( 2

2
(a cos )(b cos )
(ACB ) =
2
(b sen )(a sen )
(ADB ) =
2

)))

a(b cos(
2

))

Sus tuyendo en la primera igualdad obtenemos el resultado buscado.


El siguiente ejemplo muestra una aplicacin de las iden dades anteriores.
EJEMPLO 2.1
.
Calcule el valor del seno y el coseno para el ngulo 12
 =  , por lo que podemos aplicar las iden dades para la diferencia de dos
Solucin: Note que 3
12
4
ngulos y los valores ya conocidos para el seno y coseno de 4 y 3 . Para el valor del coseno:

(  )

cos = cos
3 4
12




= cos cos + sen sen
p3 4p p 3 4
2
3
2
1

+
= 
2
2
2p 2
p
2
( 3 + 1)
=
4
p
 = sen (   ) = 2 (p3 1).
De manera similar obtenemos que sen 12
4
4
3

16

2.5.3.

Iden dades de duplicacin y linearizacin

De las frmulas obtenidas en la seccin anterior, tenemos el caso especial cuando = , de donde:

cos(2 ) = cos2

sen2

sen(2 ) = 2 sen cos

Si u lizamos la relacin cos2 + sen2 = 1 obtenemos:

cos(2 ) = 2 cos2

cos(2 ) = 1

2 sen2

De donde es posible expresar al cos2 y al sen2 en funcin de cos(2 ):

cos2 =

1 + cos(2 )
2

sen2 =

cos(2 )
2

Tambin podemos demostralas geomtricamente u lizando la siguiente gura:

En el crculo unitario se construye el tringulo ABC con AB dimetro y C un punto cualquiera sobre
la circunferencia. Trazamos la altura respecto a AB y as los tringulos ABC y ACD son semejantes
(criterio A-A). Planteando las proporciones entre sus lados homlogos tenemos:

CD AC AD
=
=
CB AB AC
Si tomamos

CD AC
=
CB AB
sen(2 ) 2 cos
=
2 sen
2
sen(2 ) = 2 sen cos
Y si tomamos

AC AD
=
AB AC
2 cos 1 + cos(2 )
=
2
2 cos
cos(2 ) = 2 cos2 1
17

EJEMPLO 2.2
Calcule el seno y coseno de 8 .

Solucin. Ya que conocemos el valor del coseno de 2 


= , podemos u lizar las iden dades de
8
4
p
linearizacin con =  . Ya que cos  = 2 :
8

 1+
cos2 =
8
2

p2
2

2+ 2
=
4

 1
sen2 =
8
2

p2
2

La posicin en el crculo unitario del punto asociada con 8 muestra que cos 8 y sen 8 son nmeros
posi vos. En consecuencia,


cos =
8

2.6.

2+ 2
2


sen =
8

Tabla resumen de iden dades trigonomtricas ms importantes

sen2 + cos2 = 1
1 + tan2 = sec2
1 + cot2 = csc2
sen( + ) = sen cos + cos sen
sen( ) = sen cos cos sen
cos( + ) = cos cos sen sen
cos( ) = cos cos + sen sen
tan + tan
tan ( + ) =
1 tan tan
tan tan
tan ( ) =
1 + tan tan
cos ( ) cos ( + )
sen sen =
2
cos ( + ) + cos ( )
cos cos =
2
sen ( + ) + sen ( )
sen cos =
2
sen(2 ) = 2 sen cos
2
cos(2 ) = cos2 sen
= 1 2 sen2 = 2 cos2
( ) 1 cos
sen2
=
2)
2
(
1
+
cos
cos2
=
2 (
2)
(
)
A+B
A B
sen A + sen B = 2 sen
cos
( 2 )
( 2 )
A+B
A B
sen A sen B = 2 cos
sen
( 2 )
( 2 )
A+B
A B
cos A + cos B = 2 cos
cos
( 2
)
( 2
)
A+B
A B
cos A cos B = 2 sen
sen
2
2
18

Iden dad Pitagrica

Iden dad del Seno de una Suma

Iden dad del Coseno de una Suma

Iden dad de la Tangente de una Suma

Iden dad del Producto de Senos


Iden dad del Producto de Cosenos
Iden dad del Producto Seno-Coseno

Iden dad de ngulo Doble

Iden dad del ngulo Mitad

Iden dad de la Suma de Senos


Iden dad de la Resta de Senos
Iden dad de la Suma de Cosenos
Iden dad de la Resta de Cosenos

2.7.

Lecturas y ejercicios complementarios

2.2.1.- MINED(2014). Leccin 5: Trigonometra. En Material de Autoformacin e Innovacin Docente


para Matem ca 1 y 2 ao de Bachillerato. (pp.70-82). Viceministerio de Ciencia y Tecnologa:
El Salvador.

2.8.

Problemas

PROBLEMA 2.1
Ahora considere que el punto P se rota con centro el origen 90 , 180 y 270 , generando los puntos Q,
R, y S , respec vamente. Por otra parte, las reexiones del punto P con respecto al eje x y al eje y son
respec vamente U y T . Finalmente, se construye el punto V que es la reexin de P con respecto a la
recta y = x .

1. Calcule las coordenadas de Q, R, S , T , U y V en trminos de sen y cos . (U lice congruencia


de tringulos.)
2. Demuestre que si + = 180 (suplementarios) entonces sen = sen , cos =
tan = tan .

cos y

3. Si en cambio, si + = 90 (complementarios) entonces sen = cos , cos = sen y


tan = cot .
4. Mientras que si =

entonces sen =

sen , cos = cos y tan =

tan .

5. Determine los valores de seno y coseno de los ngulos de las forma siguiente (k

k k k
,
,
.
6 4 3

19

2 Z): 2k, k
,
2

1
6. Resuelva sen (3x ) = , cos (4x ) =
2

7. Complete la siguiente tabla en trminos de sen y cos


+ 

sen

cos

cos

tan

tan

cot

+  =
2
(
)=
cot +

sec

sec

csc

csc

sen

(
(

+ 3

+
)
2

= cos

)=
2

sen (

+ ) =

sen

sen

+ ) =

cos

+ ) =

tan

sec (

+ ) =

sec

csc (

+ ) =

csc

2
2

2 )
3

cos

)
+  =

+ ) =

tan (

)=

+
=
2 )
+ 3 =
2 )
(
3
cot +
=

cos (

cot (

2 )
3

)
+ 3 =
2

sen (

 ) = sen

sen (

) =

sen

sen

 ) =

cos (

) =

cos

 ) =

tan (

) =

tan

 ) =

cot (

) =

cot

sec (

 ) =

sec (

) =

sec

csc (

 ) =

csc (

) =

csc

cos (
tan (
cot (


(
2

(
2

(
2

(
2

(
2

(
2

= cos
=

)
)

=
=

PROBLEMA 2.2
Sean P y Q dos puntos sobre la circunferencia. Sean A y B las proyecciones respec vas de P y Q sobre
el eje x . Sea C la proyeccin de Q sobre OP , mientras que D y E son las proyecciones de C sobre el eje
x y QB , respec vamente.

Si \AOP = y \P OQ = , demuestre que


1. OC = cos y CQ = sen .
2. CD = sen cos = EB .
3.

\BOC = \BQC .

4. QE = cos sen .
5. QB = sen( + ) = sen cos + cos sen .
6. Dado que sen(
cos sen .

) = sen ( + ( )) , deduzca la frmula sen(

)

x , demuestre que cos( + ) = cos cos sen sen
2
) = cos cos + sen sen .

7. U lizando la relacin cos x = sen


y cos(

) = sen cos

20

Resolucin de tringulos y
ecuaciones trigonomtricas

1. Aprendizaje inicial de la Lengua es

21

Para la discusin de esta seccin, usaremos las siguientes notaciones y convenciones para un tringulo
ABC dado:

a, b, c : las longitudes de los lados opuestos a los vr ces


A, B y C , respec vamente.

S : el rea del tringulo

cos Ab y sen Ab: el cos y sen , donde es la medida


entre 0 y  de \BAC .

a
B

Teorema 3.1. (Ley del Coseno). En un tringulo ABC cualquiera, se cumple:

c 2 = a2 + b 2

2ab cos 

Demostracin. Dado el tringulo ABC , trazamos una circunferencia


de radio a y centro en B . Tomamos el dimetro CF y un punto G
cualquiera sobre la circunferencia para formar el tringulo rectgulo
CF G . Tenemos entonces que CG = 2a cos  y GA = 2a cos  b.
Ya que las cuerdas CG y DE son secantes tenemos que:

G
2acos

D
a

A
c

CA  AG = DA  AE
b  (2a cos  b) = (a c )  (a + c )
2ab cos  b2 = a2 c 2
c 2 = a2 + b2 2ab cos 

Teorema 3.2. (Ley de los Senos). En un tringulo ABC cualquiera, se cumple:

abc
c
b
a
=
=
=
b
b
b
2S
sen A sen B sen C

3.1.

Ecuaciones trigonomtricas

Como se mencion anteriormente, a diferencia de las iden dades, que se cumplen para cualquier valor
de las variables en cues n, las igualdades en las ecuaciones se cumplen slo para algunos valores de
las variables y resolverlas implica encontrar dichos valores. En ecuaciones trigonomtricas, la principal
estrategia es u lizar las iden dades antes vistas para simplicar lo ms posible la igualdad. Por ejemplo,
para la siguiente ecuacin:

2 sen(x )

1=0

Al sumar 1 a ambos lados de la ecuacin y luego dividir por 2 obtenemos:

sen(x ) =
22

1
2

De los valores obtenidos para sen(x ) en la seccin 2.4 sabemos que x = 6 . Sin embargo, al resolver una ecuacin
deben obtenerse todos los valores de la variable que hacen
que la igualdad sea verdadera. Para el caso del seno, que en
el crculo trigonomtrico equivale a la coordenada y del punto sobre el crculo que describe al ngulo, podemos observar
en la gura 10 que x =  6 = 56 tambin cumple con la
igualdad4 . Es decir, tenemos que


x=
6

Figura 10

5
x=
6

Pero hay que recordar, adems, lo discu do en la seccin 1.2: si desde cualquier punto del crculo trigonomtrico se realiza una rotacin completa, se regresa al mismo punto de par da, por lo que el seno del
ngulo descrito originalmente y el del ngulo que se forma luego de la rotacin son exactamente los mismos. As, para la primera de nuestras soluciones, sen( 6 ) = sen( 6 +2 ), de donde x = 6 +2 = 136 es
otra solucin a nuestra ecuacin. Ya que podemos repe r este mismo proceso de manera indenida (es
decir, podemos dar tantas rotaciones completas como nos plazca y siempre llegaremos al mismo punto
de par da), podemos decir que a par r de la solucin x = 6 se pueden obtener innitas soluciones de la
forma x = 6 + 2k , donde k es un nmero entero cualquiera. Aplicando esta misma idea a la segunda
de las soluciones obtenidas anteriormente, podemos decir ahora que las soluciones de la ecuacin son:

x=


+ 2k
6

x=

5
+ 2k
6

donde k es un nmero entero.


Note que el mtodo u lizado en el ejemplo anterior es similar al que se u lizara para resolver ecuaciones
con coseno o tangente y puede ser descrito en los siguientes pasos:
1. Llevar la ecuacin a la forma sen(x ) = a, cos(x ) = a, tan(x ) = a.
2. Encontrar las soluciones entre 0 y 2 de estas ecuaciones simples. Para esto puede u lizar las
igualdades:

sen(x ) = sen( x )
cos(x ) = cos( x )
tan(x ) = tan( + x )
3. U lizar la propiedad peridica de las funciones para que, a par r de una solucin especca x0 , se
pueda obtener la solucin general x = x0 + 2k , con k entero.
A con nuacin se presentan ms ejemplos que u lizan esta estrategia para su solucin.
EJEMPLO 3.1
Resuelva en R la ecuacin sen x =

p3
2

Note que lo que aqu descrito es una aplicacin de la igualdad sen = sen(
el coseno, nos hubiese sido l la iden dad cos = cos( ).
4

23

). Si la discusin se hubiese basado en

Solucin. Conocemos
(  ) una solucin par cular de esta ecuacin: x0 =
en sen x = sen
, que ene como soluciones:
3


+ 2k
3

x=
donde k

x=

4
+ 2k
3

2 Z(. Note que


) el segundo conjunto de soluciones se ob

sen = sen 2

 . La ecuacin se convierte as

ene tomando en cuenta la iden dad

EJEMPLO 3.2
Resuelva en R la ecuacin cos x = sen 5 .
Solucin. Llevaremos esta ecuacin a la forma cos x = cos x0 gracias a la relacin:
(  )

3

sen

= cos

= cos

10

Por consiguiente, resolvemos la ecuacin cos x = cos 310 , de donde tenemos la solucin

x=

3
+ 2k (k
10

2 Z)

Note que dado que cos = cos( ), tambin tenemos como soluciones

x=

3
+ 2k (k
10

2 Z)

EJEMPLO 3.3
Resuelva en [0; 2 ] la ecuacin cos 3(x = sen
) x.

Solucin.
Tenemos
que sen x = cos 2 x , de donde la ecuacin propuesta es equivalente a cos 3x =
(
)
cos 2 x . Obtenemos por lo tanto:

3x =


2

o equivalentemente:

x + 2k



+k
8
2

3x = x


+ 2k
2


+ k
4
con k 2 Z. De estos valores, aquellos que pertenecen al intervalo [0; 2 ] son:
x=

x=

De la primera relacin: 8 , 8 + 2 , 8 +  , 8 + 32 . Es decir: 8 , 58 ; 98 ; 913 .


De la segunda relacin: 

 , 2
4

 ; es decir, 3 , 7 .
4
4
4

EJEMPLO 3.4
p
Resuelva en [0; 2 ] la ecuacin 2 sen x cos x = 3 cos(x ).
Solucin. Podemos transformar la ecuacin dada a 2 sen x cos x

cos x (2 sen x
24

3) = 0

3cos (x ) = 0, de donde

Esto es cierto cuando cualquiera de los dos trminos que se estn mul plicando son iguales a 0, por lo
que analizamos estos casos por separado. Para cos x = 0, podemos observar
pque en intervalo dado los

3
valores que cumplen con la ecuacin
son x = 2 x = 2 . Para 2 sen x
3 = 0, transformamos la
p3

ecuacin para tener sen(x ) = 2 sen(x ) = sen 3 . Dentro del intervalo dado, el nico otro valor que
cumple con esta condicin es x =  3 = 23 .
Combinando los dos casos anteriores, tenemos que las soluciones de la ecuacin son 3 , 2 , 23 , 32 .
3.1.1.

Ecuaciones de la forma A cos (x ) + B sen (x ) = C

Note que en todos los ejemplos anteriores se han resuelto ecuaciones de la forma

A cos (x ) + B sen (x ) = C
Por lo que ahora describiremos un mtodo general para la solucin de dichas ecuaciones. Para esto,
suponga que un tringulo rectngulo ene por catetos
p A y B, los coecientes de sen (x ) y cos (x ) en
la ecuacin dada; su hipotenusa ser entonces A2 + B 2 . Dividimos toda la ecuacin por este valor,
resultandonos

A
B
p
cos
(
x
)
+
sen (x ) =
A2 + B 2
A2 + B 2

C
:
A2 + B 2

Los coecientes en esta nueva ecuacin son coordenadas de un punto en la circunferencia unitaria y por
lo tanto se pueden iden car con el seno y coseno del ngulo  asociado a dicho punto. Existen dos
posibilidades, de acuerdo a qu coeciente ser considerado abcisa y cul de ellos como la ordenada.
Por ejemplo, si hacemos

A
A + B2
B
sen  = p 2
A + B2

cos  =

la ecuacin puede ser escrita en la forma:

cos () cos (x ) + sen () sen (x ) =

C
A + B2
2

De donde las soluciones de la ecuacin original se reducen a las soluciones de la ecuacin

cos (

x) =

C
A + B2
2

Note que esto nos dice que para que la ecuacin original tenga solucin debe cumplirse

1

C
A + B2
2

1

Note que la seleccin del sen  y cos  fue completamente arbitraria. Tambin pudo haberse establecido
que

sen  =

p
25

A
A + B2
2

cos  =

B
A + B2
2

En cuyo caso la ecuacin original se transforma en

sen () cos (x ) + cos () sen (x ) = sen( + x ) =

C
A2 + B 2

EJEMPLO 3.5
Resuelva en [0; 2 ] la ecuacin cos (x ) + sen (x ) = p12 .
Solucin. Note que en esta ecuacin, A = 1, B = 1 y C = p12 . De aqu que

1
cos  =
=
2
( 1) + (1)2
sen  =

p1 =
2

p
2

1
1
2
p
=
=
2
2
( 1)2 + (1)2

Dentro de [0; 2 ] se enen dos opciones para  en la segunda ecuacin: 4 y 34 . De stas, slo 34 cumple
con la primera ecuacin, por lo que la ecuacin original se convierte en:
(
)

cos

de donde 34

3.2.

x = 3 34

x=

3
4

 ; es decir, x = 5
3
12

1
2

.

13
12

Lecturas y ejercicios complementarios

2.2.1.- MINED(2014). Leccin 6: Ecuaciones Trigonomtricas. En Material de Autoformacin e Innovacin Docente para Matem ca 1 y 2 ao de Bachillerato. (pp.83-104). Viceministerio de Ciencia
y Tecnologa: El Salvador.

26

Transformaciones en el plano: isometras,


2. Aprendizaje de la lectura inicial

traslaciones y rotaciones

27

Las transformaciones geomtricas son aplicaciones del plano en el plano tal que a cada punto de un
plano le hace corresponder otro punto del mismo plano. Las transformaciones geomtricas permiten
deducir una nueva gura de la primi vamente dada. El trasformado se le llama Homlogo del original.
Podemos clasicar las transformaciones en funcin del aspecto de la gura homloga respecto a la original en:
Isometras: Un movimiento o isometra es una transformacin que preserva todas las distancias
y por ello preserva el tamano y la forma (dimensiones y los ngulos). Estudiaremos la simetra
central y axial, traslaciones y rotaciones.
Isomrcas: Estas trasformaciones conservan la forma de la gura original (los ngulos) y existe
proporcionalidad entre las dimensiones de la gura original y la homloga. Estudiaremos la homotecia.

4.1.

Isometras: Simetr as.

Estudiaremos en este apartado dos pos de simetr as, las centrales y las axiales. En las primeras la simetr a ocurre respecto a un punto denominado centro, mientras que en las segundas la simetr a se da
respecto a una recta que es denominada eje de simetr a.
4.1.1.

Simetr as Centrales

Considere los puntos del plano P y un punto C que denominaremos centro. La simetr a central de centro
C es una transformacin del plano que env a cada punto P del plano en otro punto P 0 del plano de
manera tal que el punto C es el punto medio del segmento P P 0 , es decir, P C = CP 0 .
P

P0

Vamos aplicar esta transformacin a varios elementos geomtricos. Iniciemos por un segmento. Al segmento P Q le apliacremos la simetra respecto al punto C , para ello basta aplicarla a los extremos y unir
los extremos resultantes.
Q0

P0
C

Podemos notar que el nuevo segmento Q0 P 0 es congruente con el segmento P Q, ya que los 4P CQ y
0 CQ0
4P 0CQ0 son congruentes (se ene que CP = CP 0 y CQ = CQ0 y la igualdad de ngulos P[
CQ = P\
criterio LAL).
Veamos que sucede con un tringulo.Haremos la simetra del tringulo ABC respecto al punto D. Aplicamos la transformacin a cada uno de los vr ces y obtenemos el nuevo tringulo A0 B 0 C 0
28

Es fcil probar que los tringulos ABC y A0 B 0 C 0 son congruentes ya que enen sus tres lados iguales.
Por ejemplo para probar que AB = A0 B 0 observemos que los tringulos ABD y DAB 0 son congruentes,
0 DB 0 , por tanto AB = A0 B 0 . De
[ = A\
por tener los lados BD = DB 0 y AD = DA0 y los ngulos ADB
manera anloga podemos demostrar la igualdad de los otros lados.

4.1.2.

Propiedades de las simetr as centrales.

Las simetras centrales enen la propiedad de conservar:


1. Distancias, es decir la distancia entre dos puntos y sus correspondientes imgenes es la misma, es
decir d (P; Q) = d (P 0 ; Q0 ):
2. Puntos medios. Esta propiedad signica que dados dos puntos P y Q y sus respec vas imgenes P 0 y
Q0 , el punto medio M del segmento [P; Q] ene por imagen a M 0 que es el punto medio de [P 0 ; Q0 ].
3. El alineamiento de puntos. Esto signica que si tres o ms puntos estn alineados, sus imgenes estn
igualmente alineadas.

0 Q0 R0 =
4. Los ngulos y su orientacin. Es decir, si P[
QR = , entonces P\
5. Las reas. Es decir si una gura F ene rea A, la imagen F 0 que se ob ene despus de la simetrizacin
es siempre igual a A:
Demostracin. 1. Tomando en cuenta que la distancia de d (C; P ) = d (C; P 0 ) y d (C; Q) = d (C; Q0 ) y
0 CQ0 , por el criterio LAL de congruencia de tringulos se deduce
la igualdad de ngulos P[
CQ = P\
0
que los tringulos 4P CQ y 4P CQ0 son congruentes y en consecuencia d (P; Q) = d (P 0 ; Q0 ).
Q0

P0
C

2. Como en la prueba anterior, los tringulos 4P CM y 4MCQ son respec vamente congruentes con
los tringulos 4P 0 CM 0 y 4M 0 CQ0 , respec vamente; en consecuencia P 0 M 0 = P M = MQ = M 0 Q0 ,
de donde se deduce que P 0 M 0 = M 0 Q0 , es decir M 0 es el punto medio de P 0 Q0 .
29

Q0

M0

P0

3. Sean P , Q y R tres puntos alineados, condicin que signica que P Q + QR = P R. Por la primera
de las propiedades de las simetr as centrales, ya demostrada, de conservar distancias, sabemos que
P 0 Q0 = P Q, Q0 R0 = QR y P 0 R0 = P R, en consecuencia se ene: P 0 Q0 + Q0 R0 = P Q + QR =
P R = P 0 R0 y en consecuencia P 0 ; Q0 y R0 estn alineados
4. Considere el ngulo P[
QR. Verique que el tringulo P QR es congruente con el tringulo P 0 Q0 R0 y
que en consecuencia sus ngulos correspondientes son iguales, en par cular tendremos la igualdad
0 Q0 R0 .
P[
QR = P\
Adems, las simetras centrales ene la propiedad de que la composicin con ella misma da como resultado la iden dad. Es decir si Sc es la simetr a de centro c , entonces Sc2 (P ) = I (P ) = P:; por otra parte
las simetr as centrales ene la propiedad de que el segmento[P; Q] y el segmento imagen [P 0 ; Q0 ] son
paralelos.
Se dice que un punto P es invariante por una transformacin cuando su imagen P 0 coincide con P . Se
dice tambin que el punto es un punto jo por la transformacin. En el caso de las simetr as centrales el
nico punto que permanece invariante por ella es el centro mismo de la transformacin; para cualquier
otro punto P del plano se ene que su imagen P 0 es diferente a P .
4.1.3.

Simetr as Ortogonales

Sea D una recta del plano. Llamamos simetr a axial ortogonal o reexin a la transformacin del plano
que a cada punto P del plano lo transforma en el punto P 0 que sa sface las siguientes condiciones:
1. El punto medio del segmento P P 0 pertenece a la recta D:
2. El segmento P P 0 es ortogonal o perpendicular a la recta D:
D

P0
P

30

De la denicin se puede concluir que la recta D es mediatriz del segmento P P 0 , por ello si tomamos
un punto cualquiera E sobre la recta D se cumple que P E = P 0 E .

Apliquemos esta transformacin a un segmento cualquiera. Dado el segmento P Q lo reejaremos respecto a la recta D. Se verica que el segmento resultante P 0 Q0 es de igual longitud que el segmento

PQ

Veamos que ocurre si reejamos un tringulo cualquiera ABC respecto a la recta D.

El tringulo resultante A0 B 0 C 0 es congruente con el tringulo ABC


4.1.4.

Propiedades de las simetr as ortogonales

Las propiedades son similares a las propiedades de las simetr as centrales, salvo que la propiedad de
conservacin de los ngulos se reduce slo a la conservacin de la medida de los ngulos, no as la
orientacin de los mismos que en las simetr as ortogonales la orientacin se invierte. Todas las dems
propiedades son idn cas.
31

Una gura se dice que ene un eje de simetra ortogonal de eje D, cuando la gura queda invariante al
aplicarle la simetr a. De las guras usuales en geometr a las siguientes poseen eje de simetr a:
1. Los rombos. Sus ejes de simetr a son sus diagonales.
D

SD (P ) = R, SD (Q) = Q, SD (R) = P y SD (S ) = S .
SD0 (P ) = P , SD0 (Q) = S , SD0 (R) = R y SD0 (S ) = Q.
2. El tringulo 4ABC issceles en A ene como eje de simetr a la altura que sale de A.
3. El tringulo equiltero ene sus tres alturas como ejes de simetr a.
4. El rectngulo admite como ejes de simetr a a las rectas que unen los puntos medios de sus lados
paralelos.
5. El cuadrado admite como ejes a sus diagonales y a las rectas que unen puntos medios de sus lados
paralelos.
R

S
E1

D1

E2
D2
Q

En este caso, por ejemplo, se enen las siguientes correspondencias:


SD1 (Q) = Q, SD1 (S ) = S , SD1 (R) = P y SD1 (P ) = R.
SE1 (Q) = R, SE1 (S ) = P , SE1 (R) = Q y SE1 (P ) = S .
32

Las simetras ortogonales adems enen, como las simetras centrales, la propiedad de que la compo2
sicin con ella misma es la iden dad; es decir que SD
(P ) = I (P ) = P ; sin embargo, en las simetr as
ortogonales, un segmento [P; Q] y su segmento imagen [P 0 ; Q0 ], en general, no son paralelos.
En las simetr as ortogonales los puntos jos son todos los puntos que pertenecen a la recta de simetrizacin.
4.1.5.

Algunas ac vidades para realizar en el aula

Ac vidad 1: Con esta ac vidad el alumno podr descubrir que ocurre al reejar un tringulo respecto a
una recta.
1. Dobla una hoja de papel. Hazle tres perforaciones con un aller, marcando estas con las letras A,
B y C y vuelve a desdoblarla.
2. Une A con A0 ( con lnea punteada y na ) ;( A0 es el punto imagen de A resultante de la perforacin
del aller );B con B 0 y C con C 0 .
3. Une A con B y C . Estas con lnea entera. Tambin une A0 con B 0 y con C 0 . Resultan dos tringulos.
Colorea los tringulos resultantes
4. Mide el segmento desde A hasta la lnea de doblez y desde esta hasta A0 . Igual con B y C . que
sucede ?
5. Qu se puede decir del segmento AA0 con respecto al doblez?
Ac vidad 2: Con esta ac vidad se prentende que el alumno iden que la simetra de algunas guras.
1. Doble una hoja de papel tamano carta de tal manera que forme un cuadrado con la mayor rea
posible
2. Ahora, elimine la parte de papel que sobra recortndolo, dejando as slo el cuadrado
3. Luego, doble el cuadrado juntando dos esquinas opuestas. Repita el procedimiento con las otras
dos esquinas opuestas. Marque bien ambos doblados y formar las diagonales del cuadrado
4. A con nuacin, junte las diagonales de manera que forme un tringulo
5. Junte las diagonales del cuadrado en un lado de un tringulo.
6. Luego, con una jera, haga cortes en lados del tringulo que no sean los lados del cuadrado.
7. Tenga presente que los cortes sern duplicados dependiendo de los dobleces que haya realizado.
8. Qu propiedad enen las guras geomtricas que observa en el papel producto de los cortes?
Ac vidad 3: Encuentre la imagen de cada gura aplicando simetra central o axial.

33

34

4.2.

Traslaciones.

Una traslacin desliza la gura a lo largo de una trayectoria recta, moviendo cada punto la misma distancia en la misma direccin. Se puede describir una traslacin usando un vector de traslacin, que
especica tanto la distancia como la direccin.
Denicin: Sea !
u un vector, se llama traslacin del vector !
u a la asociacin de los puntos M y M 0 del
!
plano tales que: MM 0 = !
u.
t!u
!
0
0
En general se denota t!
u a la traslacin del vector u , y escribimos: M = t!
uM oM !M
Para ilustrar esta aplicacin u lizaremos el plano cartesiano. Tenemos el tringulo CDE al cual le aplicaremos la traslacin del vector u~.

El vector u~ nos indica que relizaremos un desplazamiento de cinco unidades hacia la derecha y tres
unidades hacia arriba. Si anclamos el vector u en el origen lo podemos representar u = (5; 4). As por
ejemplo la imagen del punto C (2; 2) la encontramos desplazando este punto cinco unidades hacia la
derecha y tres unidades hacia arriba obteniendo el punto C 0 (7; 5), de manera anloga encontramos las
imagenes de los otros dos vr ces:

tu~ (C (2; 2)) = C 0 (7; 5)


tu~ (D(4; 6)) = D0 (9; 9)
tu~ (E (9; 2)) = E 0 (14; 5)

35

4.2.1.

Ac vidad

Traslade cada gura segn el vector dado.

4.2.2.

Propiedades de las Traslaciones.

Las traslaciones poseen las siguientes propiedades:


La imagen de una recta, es otra recta paralela a ella.
La imagen de un segmento, es otro segmento de igual longitud.
La imagen de una circunferencia de centro O, es otra circunferencia del mismo radio, cuyo centro
es la imagen de O por la traslacin.
Conservacin del alineamiento de puntos.
36

Dado que la imagen de una recta es otra recta, tambin se cumple que si tres puntos A; B; C estn
alineados, las imgenes respec vas A0 ; B 0 ; C 0 estn tambin alineados.
Conservacin del paralelismo.
Si d1 y d2 son dos rectas paralelas, las imgenes respec vas d10 y d20 son paralelas tambin.
De esta propiedad es fcil deducir que: La imagen de un paralelogramo, es otro paralelogramo .
Conservacin de las distancias y las reas.
La imagen de un segmento, es tambin un segmento de la misma longitud.
La imagen D0 de una supercie D, ene la misma rea que D.
Conservacin del punto medio de un segmento.
Sea I el punto medio del segmento AB , si A0 B 0 es la imagen del segmento AB , entonces I 0 es la
imagen de I , adems I 0 es el punto medio del segmento A0 B 0 .
Conservacin de la medida de ngulo y de la ortogonalidad.
0 A0 y 0 del ngulo xAy
d ene la misma medida que xAy
d.
La imagen x\

En par cular cuando dos rectas d1 y d2 son perpendiculares sus respec vas imgnes d10 y d20 son
tambin perpendiculares.
EJEMPLO 4.1
!
A, B , C son tres puntos tales que C es la imagen de B por la traslacin de AB . Haga una gura de esta
situacin y responda Cul es el punto medio de AC ?

Demostracin. !
u = AB luego por denicin tu (A) = B , y de acuerdo a las propiedades de las traslaciones la imagen de AB es otro segmento de la misma longitud, y como AB = BC , entonces B es el
punto medio de AC .
A

EJEMPLO 4.2
4ABC es un tringulo cualquiera. Denotaremos por B0 a la imagen de B por la simetra central de
centro A. Trazamos la recta d paralela a AB y que pasa por C , y la recta  paralela a BC y que pasa
por A. Las rectas d y  se cortan en C 0 . Demostrar que el tringulo 4B 0 AC 0 es la imagen del tringulo
4ABC por traslacin.
Demostracin. De acuerdo a la forma del 4ABC (Ver gura), transformamos el 4ABC en el 4B 0 AC 0
! !
!
por la traslacin del vector BA hacia A0 B 0 o CC 0 . Por lo cual, ser suciente demostrar que tu (A) = B 0 ,
tu (B ) = A y tu (C ) = C 0 .
37

tu (B ) = A, este resultado se ob ene de la denicin de traslacin del vector !


u.
tu (A) = B 0 , en efecto, B 0 es la imagen de B , por la simetra central de centro A. Como A es
!
! u y esto por denicin es
!
el punto medio de BB 0 , se ene que BA = AB 0 . Es decir AB 0 = !
tu (A) = B 0 .
tu (C ) = C 0 , en efecto, los lados opuestos del cuadriltero AC 0 CB son dos a dos paralelos, es decir
! !
! u , lo cual signica que t (C ) = C 0.
se trata de un paralelogramo, esto es: CC 0 = BA =) CC 0 = !
u
d
B0
C0


B

EJEMPLO 4.3
Dado el 4ABC sea H su ortocentro. BCDE es un rectngulo construido exteriormente. Se trazan,
por D la perpendicular a AB , y por E la perpendicular a AC . Estas dos rectas se cortan en I . U lice la
! !
traslacin t!
u del vector u = EB para demostrar que A, H e I estn alineados.

Demostracin. Dado !
u = EB , por denicin tu (E ) = B , y dado que BCDE es un rectngulo tambin se cumple tu (D) = C . Como la imagen por una traslacin de una recta es otra recta paralela a la
primera, la recta BM es la imagen por !
u de la recta ER, dado que al ser ambas perpendiculares a AC
resulta inmediato que ER k BM . Anlogamente, la imagen de la recta DS por !
u es la recta CP .
La interseccin de ER y DS es I , como la traslacin es biyec va (en el sen do de que es punto a punto) sta lleva intersecciones de rectas a intersecciones de rectas, luego t!
u (I ) = H y esto implica que
!
!
IH ? BC (dado que IH = u por denicin de traslacin, y este l mo vector evidentemente es perpendicular a BC dada la forma cmo se deni). Como H es el ortocentro del 4ABC , AH ? BC . As
IH k AH (ambas son perpendiculares a BC ) y esto obliga a que A, H , I estn alineados.
A
P

M
H
R

B
I
E

38

4.3.

Rotaciones.

4.3.1.

Rotacin en el plano orientado.

Un ejemplo es l ahora para entender la importancia de trabajar en el plano orientado. En este caso
el centro de rotacin es O y el ngulo es 2 . En la gura adjunta, adems de la ortogonalidad indicada,
suponga que, OM2 = OM1 = OM . La formulacin: Un punto M asociado a M 0 talque OM 0 = OM y
\MOM 0 = 2 es ambigua, dado que los puntos M1 y M2 cumplen con la condicin.

M2

M1

En cambio, en el plano orientado, la formulacin: Un punto M asociado a M 0 tal que OM 0 = OM y


! !
(OM; OM 0 ) = 2 no deja ninguna duda: un solo punto cumple con la condicin, M 0 = M1 . Por lo que
en adelante, se u lizar el plano orientado.
Denicin 4.1. M 0 es la imagen de M (M =
6 O) por la rotacin de centro O y ngulo si: OM 0 = OM
!
!
y (OM; OM 0 ) = .
M0

Notacin: R O es la rotacin de centro O y ngulo .

4.3.2.

Ac vidad

Rota cada gura respecto al punto indicado y el ngulo dado.

39

 = 75

 = 60

cccc

 = 45

 = 90

 = 120

 = 145

Puntos Invariantes:

Si = 0 entonces todos los puntos son invariantes.


Si 6= 0 entonces el centro O de la rotacin es el nico punto invariante.
Biyec vidad y Rotacin Inversa: R O es biyec va y la rotacin inversa es la rotacin de centro O y de
ngulo .
40

Demostracin. Basta demostrar que: R O (M ) = M 0 si y slo si RO (M 0 ) = M .


4.3.3.

Propiedades Esenciales

Teorema 4.1. La rotacin conserva distancias y ngulos.


1. La rotacin enva a un segmento AB a otro A0 B 0 , igual en longitud, siendo A0 imagen de A, B 0
imagen de B . Mas an el segmento AB se transforma en el segmento A0 B 0 .
2. La rotacin transforma a una recta en otra, siendo el ngulo entre ellas .
Demostracin. Sea A0 = RO; (A) y B 0 = RO; (B ), entonces, OA = OA0 , OB = OB 0 y \A0 OB 0 =
\AOB0 \AOA0 = \AOB0 \BOB0 = \AOB, as por LAL el 4A0OB0 es congruente con 4AOB,
por tanto, la longitud del segmento AB es la misma que la del segmento A0 B 0 .
A con nuacin demostramos que la imagen del segmento AB es el segmento A0 B 0 . Sea C un punto
del segmento AB o su prolongacin, y C 0 = RO; (C ), se demuestra como antes que A0 C 0 = AC
y B 0 C 0 = BC , por consiguiente A0 C 0 + B 0 C 0 = AC + CB = AB = A0 B 0 , dado que A, B y C estan
alineados, por lo tanto, A0 C 0 + C 0 B 0 = A0 B 0 lo que demuestra que A0 , B 0 y C 0 alineados. En consecuencia,
el segmento A0 B 0 es la imagen de AB y la recta A0 B 0 es la imagen de la recta AB .
Ahora, determinamos el ngulo entre la recta y su imagen. Dena D como la interseccin de las rectas
AB y A0 B 0 . Por la congruencia de los tringulos AOB y A0 OB 0 , se ene que \BAO = \B 0 A0 O y por
ende el cuadriltero ADA0 O es cclico, por consiguiente, si E es un punto sobre la recta AD entonces,
\EDA0 = \AOA0 = .

C0

A0

B0

O
Corolario. Suponga que: A0 = RO; (A), B 0 = RO; (B ) C 0 = RO; (C ), demuestre que:
La imagen del punto medio de AB es el punto medio de A0 B 0 .

La rotacin transforma a un crculo de centro C y radio r en otro de igual radio y centro C 0 .

Si AB es dimetro del crculo de centro C , entonces A0 B 0 es el dimetro del crculo de centro C 0 .


La rotacin transforma un tringulo ABC en otro igual A0 B 0 C 0

La rotacin transforma a un ngulo en otro igual e igualmente orientado.


Dos rectas ortogonales ene por imagen dos rectas ortogonales.
41

Dos rectas paralelas enen por imagen dos rectas paralelas.


Demostracin. Dejamos esta prueba como ejercicio.
EJEMPLO 4.4
Considere dos tringulos rectngulos issceles ABC y AB 0 C 0 como en la gura adjunta. Demuestre que
BB 0 = CC 0 y que las rectas BB 0 y CC 0 son ortogonales.
Solucin. Dado que por hiptesis AB = AC , AB 0 = AC 0 y el \BAC = \B 0 AC 0 = 2 , entonces,
C = RA; 2 (B ) y C 0 = RA; 2 (B 0 ). As aplicando las propiedades de conservacin de distancias y ngulos
de las isome as se concluye que BB 0 = CC 0 y como el ngulo de rotacin es 2 , BB 0 ? CC 0 .

B0

A
C0

EJEMPLO 4.5
U lizacin de una rotacin Sea ABCD un cuadrado de centro O. M es un punto del segmento AB , y
N es un punto del segmento AD tal que DN = AM . Considere la rotacion de centro O y de ngulo 2
en sen do posi vo. Demuestre que OM = ON y el \MON = 2 .
Solucin. Recuerde que en todo cuadrado las diagonales son mediatrices mutuamente, as que OA =
OD y el \AOD = 2 , en consecuencia D = RO; 2 (A), de la misma forma se verica que: A = RO; 2 (B ),
as por las propiedades de conservacin de la rotacin la imagen del segmento AB se transforma en el
segmento DA, y por consiguiente M 0 = RO; 2 (M ) es el punto del segmento DA tal que DM 0 = AM ,
por lo tanto M 0 = N , por denicin de rotacin se ene que OM = OM 0 = ON y el \MOM 0 =
\MON = 2 .

42

EJEMPLO 4.6
En la gura adjunta, ABCD es un rombo y los tringulos DCI y BCJ son equilteros. Dena una transformacin geomtrica y demuestre que A, I y J estn alineados.
Solucin. La presencia de dos tringulos equilteros del mismo vr ce C nos sugiere denir la rotacin
de centro C y ngulo 3 . Por esta rotacin B es la imagen de J , D la imagen de I . Ahora denimos A0
como la imagen de A, por consiguiente, el tringulo CAA0 es equiltero. Esto demuestra que B , D y
A0 estn alineados, dado que pertenecen a la mediatriz de AC , puesto que: BA = BC , DA = DC y
A0 A = A0 C . Ahora solo falta considerar la rotacin inversa de centro C y ngulo 3 para demostrar que
J = RC; 3 (B ), I = RC; 3 (D), A = RC; 3 (B ) estn alineados las propiedades de conservacin de
colinealidad de las isometras.

J
D

EJEMPLO 4.7
Dada
circunferencia de centro C y radio CO. Demuestre que: Si 0 = RO; ( ), P 2 , Q 2
una
0 y P 0 = RO; (P ) entonces, P , Q y P 0 estn alineados.
Solucin. Por propiedades de conservacin de isometras sabemos que 0 es la circunferencia de centro
C 0 = RO; (C ) y radio C 0 O, ya que O es el punto invariante de la rotacin. Adems, dado que las rotaciones conservan ngulos se ene que el \P CO = \P 0 C 0 O = 2, entonces, el \OQP = 180  por
ser inscrito en la circunferencia cuyo ngulo central es \P CO, y el \P 0 QO =  por ser inscrito en la
circunferencia 0 cuyo ngulo central es \P 0 CO . Por consiguiente, \OQP + \P 0 QO = 180  +  =
180 y por lo tanto, P , Q y P 0 son colineales.
43

C0

2
1800

2
P0


Q

4.4.

Transformaciones en el plano: composicin de simetras y homotecias

En est seccin:
Encontrars la transformacin simple equivalente a la composicin de dos traslaciones.
Encontrars la transformacin simple equivalente a la composicin de las reexiones a travs de
dos rectas paralelas
Encontrars la transformacin simple equivalente a la composicin de las reexiones a travs de
dos rectas que se intersecan
Cuando se aplica una transformacin a una gura y luego se aplica otra transformacin a su imagen, la
transformacin resultante se llama una composicin de transformaciones.

4.5.

Ac vidad 1

Reexiones a travs de dos lneas paralelas. En primer lugar, consideremos el caso de lneas paralelas
para la reexin
1. En una hoja de papel vegetal o un acetato, dibuja una gura y una lnea de reexin pero que no
se corten.

2. Doble el papel por la lnea para reejar su gura, marque la gura resultante.
44

3. En el papel vegetal o en el acetato, dibujar una segunda recta de relexin paralela a la primera, tal
que la imagen se encuentre entre las dos rectas paralelas.

4. Reeja la imagen respecto a la segunda recta y traza la segunda imagen.

5. u transformacin nica llevara la gura original a la imagen nal? (Sugerencia: mo se compara


la orientacin de la imagen nal con la orientacin de la original?)

Compara la distancia entre las rectas paralelas con la distancia entre un punto en la gura original y el
punto correspondiente en la imagen nal.
Usa tus descubrimientos para completar esta conjetura:

45

4.6.

Ac vidad 2

Reexiones a travs de dos rectas que se intersecan. A con nuacin, se estudiar el caso de rectas
rectas que se intersectan para la reexi.
1. En una hoja de papel vegetal o un acetato, dibuja una gura y una lnea de reexin pero que no
se corten.

2. Dobla el papel por la lnea para reejar su gura, marque la gura resultante.

3. En el papel vegetal dibuja una segunda recta de reexin que se corte con la primera, de manera
tal que la imagen forme un ngulo agudo entre la interseccin de las lineas de reexin.

4. Reeja la imagen respecto a la segunda recta y traza la segunda imagen

5. Dibuja dos semirrectas p y q que comiencen en el punto de interseccin de las dos rectas y que pasen por los puntos correspondientes de la gura original y su segunda imagen. u transformacin
simple llevara la gura original a la imagen nal?

46

Debes haber encontrado que las dos reexiones son equivalentes a una rotacin simple. Usa un transportador para comparar el ngulo de rotacin (es decir, el ngulo creado por las semirrectas p y q ) con
el ngulo agudo formado por las rectas que se intersecan.

Usa tus descubrimientos para completar esta conjetura:

4.7.

Ac vidad 3

Ahora veremos un ejemplo de composicin de dos traslaciones.


El tringulo ABC con vr ces en los puntos A( 1; 0), B (4; 0) y C (2; 6), en primer lugar es trasladado
por el vector u~ = ( 6; 5) y despus su imagen, el tringulo A0 B 0 C 0 es trasladado por el vector ~
v = (6; 3).
Qu traslacin simple es equivalente a la composicin de esas dos traslaciones?

47

Podemos observar que al componer las dos traslaciones resulta otra traslacin cuyo vector es el w
~ =

~v
u+

Ahora pasaremos a formalizar los resultados anteriores.

4.8.

La isometra directa ms simple

Teorema 4.2. Dados dos segmentos congruentes AB y A0 B 0 , existe una isometra directa que transforma
a A en A0 y a B en B 0 . Adems, de todas las isometras que cumplen esta condicin, las ms simple
siempre ser una rotacin, o una traslacin en caso excepcional.

Demostracin. Si ABB 0 A0 forma un paralelogramo, claramente la traslacin de vector AA0 transforma


A en A0 y B en B 0 . Si en cambio ABA0 B 0 forma un paralelogramo, tomando O como la interseccin de
AA0 con BB 0 , una rotacin de centro O y ngulo 180 funciona.5
A0

B0

O
A
5

Observe que esta rotacin es una reexin puntual respecto a O.

48

En cualquier otro caso (es decir, AB , A0 B 0 ), sea P la interseccin de AB con A0 B 0 , y sea O la otra
interseccin de los circuncrculos de los tringulos 4AA0 P y 4BB 0 P . Note que OP AA0 y OP BB 0 son
cuadrilteros concclicos, entonces \OAB = \OA0 B 0 y \OBA = \OB 0 A0 , y dado que por hiptesis
AB = A0 B 0 se concluye que 4ABO  4A0 B 0 O y enen igual orientacin. Note adems que si llamamos al ngulo que la recta AB forma al rotarse con centro P hasta coincidir con la recta A0 B 0 , de
nuevo por los cuadrilteros concclicos se ob ene = \AOA0 = \BOB 0 . As, una rotacin de centro
O y ngulo funciona como isometra directa que transforma a A en A0 y a B en B 0 .

B0

A0

4.9.

Descomposicin de traslaciones y rotaciones como composicin de simetras


axiales

Ya se estudi el hecho que una simetra axial es una isometra indirecta, esto quiere decir que a pesar
que man ene invariante las distancias cambia la orientacin de los ngulos. En par cular, si los vr ces
de un tringulo 4ABC se recorren en sen do an horario, los vr ces del tringulo imagen tras una
simetra axial 4A0 B 0 C 0 se recorren en sen do horario. Si ahora se aplica otra simetra axial (o la misma
reexin axial anterior, si se quiere) al 4A0 B 0 C 0 , el tringulo imagen 4A00 B 00 C 00 cumple que sus vr ces
se leen en sen do an horario. As, el 4ABC y el 4A00 B 00 C 00 son tringulos congruentes y con la misma
orientacin, esto signica que existe una isometra directa que transforma al 4ABC en el 4A00 B 00 C 00 .
Por otra parte, ya se sabe que una isometra directa siempre puede resumirse a una sola rotacin, o una
sola traslacin en caso excepcional. A pesar que no es evidente, las Reexiones Axiales pueden u lizarse
para generar Rotaciones y Traslaciones, lo que las convierte en las isometras ms fundamentales. Esto
signica que cualquier isometra puede escribirse como la composicin de reexiones axiales. Ms impresionante an es que la can dad de reexiones axiales necesarias es nicamente 2.
Teorema 4.3. Una Traslacin puede descomponerse como el producto de dos reexiones axiales.

Demostracin. Si la traslacin est determinada por el vector AB , al tomar dos rectas paralelas entre s
!
l1 y l2 , y perpendiculares a AB , tales que la distancia desde l1 hasta l2 es igual a la mitad de la longitud
!
!.
del vector AB , se cumple que la composicin de reexiones axiales Rl2 Rl1 es igual a la traslacin TAB
49

P2

P1

A
l

2d

Teorema 4.4. Una Rotacin puede descomponerse como el producto de dos reexiones axiales.
Demostracin. Dada la rotacin de centro O y ngulo , se construyen dos rectas l1 y l2 que pasan por
O, y que cumplen que el ngulo desde l1 hasta l2 es igual a 2 . Es rela vamente sencillo mostrar que la
composicin de reexiones axiales Rl2  Rl1 es igual a la rotacin de centro O y ngulo .
l2
B2

A2
A1

B1
l

O
A

A par r de estos dos teoremas anteriores es evidente el siguiente corolario:


Corolario. Toda Isometra Directa puede descomponerse como el producto de 2 Reexiones Axiales.
Ms an, una Isometra Indirecta o bien es una Reexin Axial o bien la composicin de una Isometra
Directa con una Reexin Axial, por lo tanto, toda Isometra Indirecta puede escribirse como la composicin de a lo sumo tres Reexiones Axiales. De aqu se deriva el siguiente resultado, que es fundamental
en la teora de Transformaciones Geomtricas.
Teorema 4.5. Toda Isometra puede escribirse como la composicin de a lo sumo 3 Reexiones Axiales.

4.10.

Problemas de Traslaciones

1. Sean ABCD un paralelogramo de centro O, la paralela a BD que pasa por A corta a la paralela
AC que pasa por D en el punto E . Cules son las imgenes de A y D bajo la traslacin del vector
!
EO?
2. Sean ABCD un paralelogramo de centro O, M y N los puntos medios de los segmentos DA y BC
respec vamente. Son verdaderas o falsas las siguientes armaciones?
50

a) Los puntos M; O y N son alineados.


b) Los segmentos AB y CD son simtricos con respecto a la recta MN .
c) La traslacin que transforma D en C , transforma M en N .
[ , transforma C en B .
d) La rotacin de centro O y ngulo AOD

e) La traslacin de vector AC es igual a la traslacin de vector BD.

! (B ) y E =
3. Considere el tringulo 4ABC issceles en A, I el punto medio de BC , D = tAC
! (D). Estudie la naturaleza de los cuadrilteros ABDC y DECI .
tBI

4. Construya un tringulo 4ABC , en seguida el punto D, simtrico de A con respecto a B , y el


!
transformado E de B por la traslacin de vector AC . Mostrar que el tringulo 4ABC es el transformado del tringulo 4BDE por una traslacin la cual debe precisarse el vector.
5. Sea 4ABC un tringulo rectngulo en B , y H la proyeccin ortogonal de B sobre AC . Se denotan
por A0 y C 0 los puntos tales que HBAA0 y HBCC 0 son dos paralelogramos. Cul es la naturaleza
del cuadriltero CAA0 C 0 ?
!
Sugerencia: U lice la traslacin de vector BH .
6. El cuadrado ABCD es de centro O y de lado a. Determine los puntos M sobre el lado AB , N sobre
el lado CD, tales que MN k BC y que minimicen la poligonal OMNB , y exprese este mnimo en
funcin de a.
! (O).
Pista: Sea O0 = TBC
7. Consdiere dos circunferencias de radios iguales de centro O, y 0 de centro O0 , que se cortan en
A y en B . Considere dos rectas paralelas l y l 0 :

l pasa por A, corta nuevamente a en P , y a 0 en P 0 ;


l 0 pasa por B , corta nuevamente a en Q, y a 0 en Q0 .
Demuestre que P P 0 Q0 Q es un paralelogramo.
8. Demuestre que la composicin de dos traslaciones es otra traslacin.

4.11.

Problemas de Simetras Centrales o Reexiones Puntuales

1. Cules son las guras simtricas de la gura siguiente con respecto a las cuatro esquinas del
rectngulo?

51

2. Sean las rectas d y d 0 secantes en O y el punto I no perteneciente a ninguna de las rectas anteriores.
a) Construya el punto interseccin A de la recta d 0 con la imagen de la recta d por la simetra
de centro I .
b) La recta AI corta a d en B . Precise la posicin de los puntos I , A y B .
c) Aplicacin: Construya un segmento de punto medio I , con uno de sus extremos en d y el otro
en d 0 .
3. Dado un tringulo 4ABC , sea M el punto medio de BC . D y E son las proyecciones ortogonales
respec vas de B y C sobre la mediana AM . U lizando propiedades de simetras puntuales aplicadas a la simetra respecto a M , demuestre que BD = CE , ie, los vr ces B y C equidistan de
la mediana AM .
4. Sea ABCD un paralelogramo de centro O. Especicar las imgenes de los puntos A; B; C y D
por S0 .
5. El tringulo 4ABC es rectngulo en A; denotamos por D = SA (B ) y E = SA (C ). Cul es la
naturaleza del cuadriltero BCDE ?
6. Si ABCD y AECF son paralelogramos, qu po de cuadriltero es BEDF ?
7. Dado un paralelogramo ABCD de centro O, se trazan dos rectas paralelas l y l 0 pasando respecvamente por A y C . Si l corta a la recta BC en E , y l 0 corta a la recta DA en F , determine la
naturaleza del cuadriltero BEDF .

4.12.

Problemas de Rotaciones

1. Dada la gura siguiente, construya las imgenes del cuadrado, tringulo y segmento obtenidos por
la rotacin de centro O y de ngulo 90 (en ambos sen dos).

2. Sean d una recta, O un punto no perteneciente a d y r la rotacin de centro O y ngulo 120 , en


el sen do directo. Construir, en el siguiente orden:
a) La proyeccin ortogonal de H de O sobre d .
b) La imagen H 0 de H por r .
c) La imagen d 0 de d por r .

52

d) Si denotamos por I el punto interseccin de d y d 0 , mostrar que O; I; H y H 0 estn sobre un


mismo crculo.
3. Sean:  un crculo de centro O, r la rotacin de centro O y ngulo de 50 . El punto A pertenece a
 , denotemos por B la imagen de A por r y por C la imagen de B por r .
a) Ilustre la situacin anterior.

b) Exprese en grados los ngulos del tringulo 4ABC .


4. Trace el segmento AB de 4cm. Ubique el punto O de tal forma que la rotacin de centro O y
ngulo 90 , muevan el punto A al punto B . Cul es la distancia del punto O a la recta AB .
5.

a) Cmo trazar los vr ces de un hexgono regular, u lizando slo el comps?


b) Aplicacin: Dados dos puntos O y A: Construir u lizando slo el comps, el punto simtrico
de A con respecto a O y el transformado A por la rotacin de centro O y ngulo 120 .

6. Sea el cuadrado ABCD de centro O en sen do directo. M un punto del segmento AD y N su


imagen por una rotacin de 90 en sen do directo y de centro O.
a) Demostrar que N pertenece al segmento AB .
\ y BON
\ y las reas de los tringulos
b) Comparar las distancias de BM y CN , los ngulos AOM
4BDM y 4CAN .

7. Dada una recta l y un punto O que no pertenece a l , demostrar que las imgenes de l al hacer
todas las rotaciones de centro O, son las tangentes a una circunferencia ja.
8. Dada una recta l y un punto O fuera de ella, se construye un tringulo equiltero 4OAB de tal
forma que A vara sobre l . Qu lugar geomtrico describe B ?
9. Sobre los lados de un n-gono regular A1 A2    An se dibujan puntos B1 ; B2 ; : : : ; Bn tal que A1 B1 =
A2 B2 =    = An Bn . Demuestre que B1 B2    Bn tambin es n-gono regular.
10. Dado un cuadrado ABCD nombrado en sen do horario, sea I un punto en su interior tal que
4DCI es equiltero, y J un punto en su exterior tal que 4BCJ tambin es equiltero. Demuestre
que A, I , J , estn alineados. Para ello:


a) Construya las imgenes de J , I , A tras la rotacin RC3 y demuestre que estas tres imgenes
estn alineadas.
b) Concluya por propiedades de rotaciones que J , I , A, estn alineados.
11. Los tringulos 4ABC y 4AB 0 C 0 son tringulos rectngulos issceles y con la misma orientacin.
Demuestre que 4ABB 0  4ACC 0 .

53

Transformaciones en el plano: homoteclas


4.

54

5.1.

Teoremas Fundamentales

Las homotecias, dicho de forma sencilla, son las transformaciones geomtricas bsicas que es ran o
encogen a las guras geomtricas, o dicho de manera un poco ms formal, dilatan o contraen a las guras geomtricas; en casos muy excepcionales (y de mucho inters), la gura geomtrica quedar del
mismo tamao. En cualquier caso, estas dilataciones o contracciones deben cumplir dos caracters cas:
Al transformar mediante una homotecia a una gura geomtrica cualquiera, la gura resultante debe
conservar las proporciones y la inclinacin de la gura original.6
Denicin 5.1. Homotecia: Dado un punto jo O y un nmero real no nulo k , la homotecia de centro O
y razn k , denotada por HO;k , es la transformacin puntual que, a cada punto M asigna un punto M 0 ,
tal que:
) Los puntos O, M y M 0 estn alineados, y
) Los segmentos dirigidos OM y OM 0 sa sfacen la relacin OM 0 = k  OM .

O equivalentemente, estas condiciones pueden abreviarse en la relacin vectorial OM 0 = k OM . El punto


O se llama Centro de Homotecia y el nmero real no nulo k se llama Razn de Homotecia.
Observaciones:
El nmero k puede tomar tanto valores posi vos como nega vos, esto se debe a que se est
trabajando con segmentos dirigidos, o con vectores si se preere.
Si k = 1, la homotecia HO;1 es la transformacin iden dad, y todos los puntos son puntos jos,
es decir, HO;1 (M ) = M para todo M ; en cambio, si k 6= 1, el nico punto jo es el centro de
homotecia O.
Si k =

1, la homotecia HO;

es la reexin puntual respecto a O, RO .

Teorema 5.1. La homotecia HO;k transforma al segmento dirigido AB en un segmento dirigido A0 B 0 , tal
que:

k AB, y
A0 B 0 = k  AB .

1. A0 B 0
2.
6

Esto es bsicamente lo que un programa de computadora hace a una imagen cuando se u liza la funcin zoom.
55

O equivalentemente, transforma el vector AB en un vector A0 B 0 tal que A0 B 0 = k AB .

Demostracin. Se muestran dos soluciones, una por la va sint ca y otra u lizando vectores:
Primera demostracin: Por la dencin de la homotecia HO;k , los puntos O, A y A0 estn alineados y
lo mismo sucede con O, B y B 0 , entonces \AOB = \A0 OB 0 ; adems, tambin por la denicin de
0
OB0
0 0
homotecia se ene OA
lal de semejanza de
OA = k = OB , entonces 4AOB ' 4A OB (por el 0criterio
0
A
B
0
0
tringulos) con razn de semejanza k , y por lo tanto \OAB = \OA B y AB = k , que son proposiciones equivalentes a A0 B 0 k AB y A0 B 0 = k  AB .

Segunda demostracin: Por la denicin de la homotecia(HO;k se cumple


que OA0 = k OA y OB 0 =
)
!
!
!
!
! !
! !
!
k OB , entonces A0 B 0 = OB 0 OA0 = k OB k OA = k OB OA = k AB .
Para completar las pruebas, si M es un punto cualquiera alineado con A y B , repi endo estas mismas
!
demostraciones, M 0 = HO;k (M ) cumple que A0 M 0 k AM y A0 M 0 = k  AM , o equivalentemente A0 M 0 =
!
k AM , por lo que M 0 est alineado con A0 y B 0 ; por lo tanto, el segmento dirigido AB se transforma punto
a punto en el segmento dirigido A0 B 0 , y lo mismo se dice con los vectores.
Corolario. La homotecia de una recta AB es otra recta A0 B 0 tal que AB k A0 B 0 ; en par cular, si el centro
de homotecia pertenece a la recta AB , entonces la recta A0 B 0 coincide con la recta AB .
Demostracin. Dejamos esta prueba como ejercicio.
Teorema 5.2. Recproco: Dado un nmero real no nulo k , toda transformacin que cumple que a cualquier segmento dirigido AB lo transforma en el segmento dirigido A0 B 0 vericando las relaciones A0 B 0 k
AB y A0 B 0 = k  AB , es una homotecia si k 6= 1, o es una traslacin si k = 1.7
Demostracin. Analizamos los casos por separado:
Caso 1: Si k 6= 1, denimos la homotecia HO;k como aquella que cumple A0 = HO;k (A); observe que
HO;k (B) = B1 tal que A0B1 k AB y A0B1 = k  AB, por lo que B1 coincide con B0.
En par cular, si k = 1 y A coincide con A0 , entonces la transformacin evidentemente es la transformacin iden dad, la
cual puede ser vista como una traslacin de vector nulo o como una homotecia.
56
7

Caso2: Si k = 1, se ene que los segmentos dirigidos AB y A0 B 0 guardan las relaciones A0 B 0 k AB y


A0 B 0 = AB , esta es justamente la denicin de traslacin. Es importante notar que este caso puede ser
visto como el comportamiento al lmite del caso anterior, si se inicia con un k 6= 1 y se dene nuevamente
la homotecia HO;k como aquella que cumple A0 = HO;k (A), y ahora se busca que k ! 1, ser necesario
que O se mueva sobre la recta AA0 alejndose innitamente de estos puntos, as las rectas AA0 y BB 0
enden a ser paralelas y por tanto los segmentos AB y A0 B 0 enden a ser paralelos e iguales.
Corolario. Una homotecia est bien denida si se conocen las imgenes de dos puntos dis ntos dados.
Demostracin. Dados los puntos A y B , si A0 = HO;k (A) y B 0 = HO;k (B ) son sus imgenes respec vas,
0 B0
entonces, por el teorema anterior, la razn de homotecia es k = AAB
, y el centro de homotecia O es la
0
0
8
interseccin de las rectas AA y BB .

5.2.

Homotecia de guras geomtricas fundamentales

A con nuacin se presentan como una serie de problemas guiados, los resultados ms importantes de la
homotecia aplicada a rectas, tringulos y circunferencias, que son las guras geomtricas fundamentales.
PROBLEMA 5.1
Homotecia de Rectas:
La homotecia de rectas se divide en dos casos, que la recta pase por el centro de homotecia, o que la
recta no pase por dicho centro.
1. Demuestre que si una recta pasa por el centro de una homotecia, la imagen de esta recta al aplicarle la homotecia es la misma recta.
Observacin: Esto no signica que los puntos no se han movido, sino que a pesar que se han movido, tanto el punto preimagen como el punto imagen pertenecen a la misma recta.
2. Demuestre que si una recta no pasa por el centro de una homotecia, la imagen es otra recta paralela a la original.
3. Ex enda estos resultados a la homotecia de planos.
4. Demuestre que la homotecia de un ngulo es otro ngulo con igual medida y orientacin; si el vrce del ngulo coincide con el centro de homotecia y la razn de homotecia es posi va, entonces
el ngulo es invariante tras la homotecia.
5. Demuestre que la homotecia de un semiplano denido por una recta, es otro semiplano denido por la imagen de la recta dada; demuestre tambin que los semiplanos restantes tambin se
corresponden.

PROBLEMA 5.2
Homotecia de Tringulos:
Note que por lo general, los segmentos dirigidos AB y A0 B 0 son paralelos pero de dis nto tamao, exceptuando el caso
cuando la homotecia es la transformacin iden dad, pero este es un caso trivial en el que A0 B 0 coincide con AB ; as, por lo
general k 6= 1 y por tanto las rectas AA0 y BB 0 son secantes.
57
8

1. U lizando el problema anterior, demuestre que la homotecia de un tringulo es otro tringulo tal
que los lados correspondientes son respec vamente paralelos. Estos dos tringulos pueden ser
coplanares, si el centro de homotecia se encuentra en el mismo plano que el tringulo original, o
se ubican en planos paralelos en cualquier otro caso.
2. Demuestre por tanto que la homotecia transforma a un tringulo en otro tringulo semejante, y
ambos tringulos enen la misma orientacin de los ngulos y la razn de homotecia k coincide
con la razn de semejanza. A este par de tringulos se les llama Tringulos Homot cos. Demuestre
adems que la razn entre las reas de los tringulos homot cos es k 2 .
3. Demuestre el recproco: Dados dos tringulos semejantes con lados correspondientemente paralelos (ie, tringulos homot cos), existe una homotecia que transforma a un tringulo en el otro.
Para ello, hay que demostrar que:
a) El centro de homotecia existe, es decir, que las tres rectas que unen vr ces correspondientes
(aquellos que se oponen a lados correspondientemente paralelos) son concurrentes.
b) La razn de homotecia es la razn de semejanza de los tringulos.
4. Ex enda la homotecia a cuadrilteros, pentgonos, etc.
5. Demuestre que la homotecia del interior de un tringulo es el interior del tringulo imagen, y
anlogamente, el exterior del tringulo se mapea al exterior del tringulo imagen. Qu sucede
con el interior y el exterior de un polgono simple9 ?

PROBLEMA 5.3
Homotecia de Circunferencias.
1. Demuestre que la homotecia de una circunferencia es otra circunferencia. Si el centro de homotecia est en el mismo plano que la circunferencia, la circunferencia resultante es coplanar a la
original; en cambio, si el centro de homotecia no est en el mismo plano que la circunferencia, la
circunferencia resultante est en un plano paralelo al plano de la circunferencia original.
2. Demuestre que si dos circunferencias dis ntas son concntricas, el centro de stas es un centro
de homotecia que transforma una circunferencia en la otra.
3. Demuestre que si dos circunferencias no concntricas enen radios iguales, el punto medio del
segmento que une los centros de las circunferencias es un centro de homotecia para stas (de
hecho, es una reexin).
4. Dadas dos circunferencias no concntricas y de radios dis ntos, si AB es un dimetro de la primera
circunferencia, A0 B 0 es un dimetro de la segunda circunferencia, y AB k A0 B 0 , demuestre que
a) El punto de interseccin de AA0 con BB 0 es un centro de homotecia de las circunferencias.

b) El punto de interseccin de AB 0 con A0 B es otro centro de homotecia de las circunferencias.


9

Un polgono simple es aquel no ene auto-intersecciones.

58

5. Demuestre que en todos los casos, la razn de homotecia es igual a la razn de los radios.
6. Demuestre que el interior de una circunferencia se mapea al interior de la circunferencia imagen,
y que el exterior se mapea al exterior.
7. Ex enda estos resultados a la homotecia de esferas.

PROBLEMA 5.4
El Pantgrafo:
La siguiente gura muestra un pantgrafo, un instrumento de diseo u lizado para aumentar o disminuir
dibujos. El pantgrafo est compuesto por 4 varillas unidas por ar culaciones, y ene las siguientes
caracters cas:
El punto O est jo, y el punto M es variable.
Las varillas OA y AM enen igual longitud r . Esto le permite a M moverse 2r a la redonda de O.
El punto B es un punto jo sobre la varilla OA, pero escogido arbitrariamente. Este punto denir
la escala.
El cuadriltero ABM 0 C es un paralelogramo de tal manera que O, M y M 0 estn alineados.

Para u lizar el pantgrafo, debe colocarse un lpiz sobre el punto M 0 ; al mover el punto M describiendo
OA . Note que puede interlos contornos de un dibujo, el punto M 0 reproduce el dibujo a una escala 1 : OB
cambiarse los roles de M y M 0 para lograr un dibujo con una escala 1 : OB
OA .
Tambin podemos construir un modelo de pantgrafo en la computadora, siguiendo los siguientes pasos:
1 En un programa de geometra, dibuje dos puntos cualesquiera, O y M .
59

2 Dibuje dos circunferencias de radio jo r , una con centro en O y otra con centro en M .
3 El problema tendr sen do nicamente si O y M estn lo sucientemente cerca como para que las
dos circunferencias se corten, si esto no sucede, acerque un poco M hacia O. Realizado esto, dena
A como una de las intersecciones de las circunferencias.
4 Dibuje un punto B sobre el segmento OA. Este punto quedar jo sobre el segmento OA, esto quiere
decir que siempre mantendr constante las distancias hacia los extremos del segmento.
5 Por B , trace una recta paralela a AM y dena M 0 como la interseccin de esta recta con OM .
6 El pantgrafo est listo, ac ve la opcin de Dejar rastro a los punto M y M 0 ; a medida que Ud.
mueva M , el programa le generar un dibujo a escala descrito por M 0 .
Observe que para este modelo terico de pantgrafo no es necesario construir el punto C , en el pantgrafo real s lo es porque no es posible ordenar a las varillas que se mantengan paralelas, entonces se
recurre a un ar cio mecnico; de cualquier forma, si Ud. desea dibunar el punto C , ene que trazar
una recta por M 0 paralela a OA, y la interseccin de esta recta con AM dene al punto C .
Finalmente, para consolidar todo lo anterior
1. Jus que el funcionamiento del pantgrafo demostrando que para todo punto M , se cumple que
M 0 = HO; OB (M ).
OA

2. Elabore un pantgrafo en la computadora y verique las propiedades de homotecia en el plano


para rectas, tringulos y circunferencias; note que previo al diseo del pantgrafo, debe tener lista
la gura a la cul se le aplicar la homotecia y obligar al punto M a pertenece a dicha gura.

PROBLEMA 5.5
Homotecia de una gura cualquiera:
En general, dada una gura geomtrica en el plano o en el espacio, es posible transformarla mediante
una homotecia y (segn las discusiones previas) la gura resultante siempre ser semejante a la original;
por ejemplo, la homotecia de un pentgono es otro pentgono que, no necesariamente es del mismo
tamao que el original, pero s ene los mismos ngulos y sus lados son correspondientemente paralelos
al original; esto mismo sucede con un arco de circunferencia, una fotogra a, un cilindro, uno cono, etc.
Sin embargo, cmo se puede llegar a una conclusin tan poderosa nicamente analizando la homotecia
aplicada a guras tan elementales como la recta, los tringulos y las circunferencias?
Para completar formalmente el argumento, es necesario recurrir al Clculo; sin entrar en tantos detalles,
en esencia, la idea es descomponer localmente cualquier curva en una lnea poligonal de lados inni amene pequeos, una supercie en un poliedro triangular de caras innitamente pequeas, teselar una
regin plana con tringulos innitamente pequeos, etc. y luego aplicar la homotecia a la poligonal (por
ejemplo) esperando que la poligonal imagen describa la curva imagen.
Aunque esto es plausible, es importante establecerlo con ms rigor; cualquiera que haya tenido experiencia con el Clculo, sabr que la idea que est de fondo es la con nuidad de la homotecia. Esta es
una propiedad, que dicho de manera sencilla establece que: dado un punto y un entorno cercano a ese
60

punto, la homotecia lo mapea en un entorno cercano a la imagen del punto. Para ello, aunque no es
nada evidente, lo que debe probarse es dado un punto P y su imagen P 0 = HO;k (P ), si se contruye
una circunferencia de radio arbitrario  y centro P 0 , siempre es posible encontrar un radio  tal que la
imagen tras la homotecia de la circunferencia de radio  y centro P est completamente contenida en
la circunferencia de centro P 0 y radio .
Otra propiedad importante es que la homotecia es una transformacin biyec va; esto resulta par cularmente l cuando se quiere medir el ngulo entre la homotecia de dos curvas: Suponga que y son
dos curvas que se cortan en un punto P , el ngulo entre y est denido como el ngulo que forman
las tangentes locales a las curvas en P ;10 suponga que 0 , 0 y P 0 son sus respec vas imgenes tras una
homotecia.
En primer lugar observe que 0 y 0 se cortan en P 0 .
En segundo lugar, y 0 pueden verse como poligonales (llevadas al lmite) que por propiedades
de homotecia, enen lados correspondientemente paralelos, entonces las curvas son localmente
paralelas.
En tercer lugar, si l es la recta tangente localmente a en P , se transforma en una recta l 0 paralela
a l , que pasa por P 0 , y si llegara a cortar (localmente) a l 0 en un segundo punto P 00 , la preimagen
forzosamente debera ser P , lo que contradice la inyec vidad de la homotecia; as, l 0 es tangente
a 0 en P 0 .
Finalmente, si m es la recta tangente a en P , la imagen m0 es la recta tangente a 0 en P 0 , por
propiedades de homotecia, el ngulo entre l y m es igual al ngulo entre l 0 y m0 , entonces, el ngulo
entre y es igual al ngulo entre 0 y 0 . Por lo tanto, una gura, por complicada que sea, cumple
que su imagen tras una homotecia es semejante a la original, preservando la inclinacin.
Para completar lo comentado en este prrafo, resuelva lo siguiente:
1. Demuestre que la homotecia es una transformacin con nua.
Se denota por C (P; r ) al interior de una circunferencia (o una esfera) de centro P y radio r . Sea P 0
el imagen de un punto P tras la homotecia HO;k . Demuestre que para un radio arbitrario  > 0, es
posible construir convenientemente un radio  > 0 tal que HO;k (C (P;  )) est completamente
contenido en C (P 0 ; ).
2. Demuestre que la homotecia es una transformacin biyec va.

EJEMPLO 5.1
RECTA Y CRCULO DE EULER DE UN TRINGULO. Demuestre que en todo tringulo, el centroide G est
situado entre el segmento OH , el cual une el circuncrculo y el ortocentro.
!
!
La recta OH es la recta de Euler del tringulo ABC y adems MO = 12 AH
10

Observe que para denir la tangente local a una curva, es necesario hacer uso de lmites, y la con nuidad de la hometecia
es nuevamente un tema central.

61

A
P

G
M

Sean M; N; P los puntos medios de los lados BC; CA y AB de un 4ABC . El 4MNP cuyos lados son
respec vamente paralelos a cada uno de los lados del 4ABC , es la transformacin a travs de una ho1
motecia de razn NP
BC = 2 .
El centro G de esta homotecia es comn con el punto donde se cortan las rectas AM , BN y CP y es
!
!
tal que GM = 12 GA, con esto se comprueba mediante homotecias la propiedad de la mediana de un
tringulo.
Acabamos de establecer la homotecia HG; 1 , observemos que la altura trazada desde A del 4ABC ,
2
ene por homlogo a la altura trazada desde M del 4MNP , y esta se dir que es la mediatriz de BC .
!
!
El ortocentro H del 4ABC ene por homlogo el centro O del cincuncrculo del 4ABC y GO = 12 GH

P
O

N
G

B
M

A0

H1
Demostremos ahora que En todo tringulo, los puntos medios de los lados, los pies de las alturas y los
puntos medios de los segmentos que unen al ortocentro con los vr ces del tringulo, estn situados en
una misma circunferencia, que llamaremos circunferencia de Euler o circunferencia de los nueve puntos.
Prueba
Sea w el centro de la circunferencia que pasa por los puntos M; N y P , este centro w es el homlogo
!
!
de O, que es el centro de la circunferencia que pasa por A; B y C y Ow homlogo de HO, de aqui que
!
!
!
!
Ow = 12 HO. Luego el punto w es el punto medio de OH . La igualdad Hw = 12 HO muestra que H es
el centro de la homotecia de las circunferencias ABC y MNP , cuya razn es posi va.
Veamos ahora que con la homotecia encontrada HH; 1 la transformacin del punto A es el punto medio
2
de AH , y la transformacin del punto H1 de la recta AH corta a la circunferencia ABC en el pie A0 de
62

la altura AH .
Los puntos y A0 pertenecen a la circunferencia MNP .

C0
O

! H
J
A0

N
B0

C

H1
5.3.

Problemas de Homotecias

1. Dos circunferencias
y
0 son tangentes en I . Se considera las cuerdas MN en la circunferencia

de longitud ja. Las rectas MI y NI cortan nuavamente a


0 en M 0 y N 0 , respec vamente.
Demuestre que la recta M 0 N 0 permanece tangente a una circunferencia ja.
2. Sean y 0 dos circunferencias tangentes en T , y sea A un punto interior a . Dado un punto variable M sobre , se dene N como la otra interseccin de AM con , luego M 0 es la otra interseccin
de MT con 0 , y N 0 es la otra interseccin de NT con 0 . Demuestre que las rectas M 0 N 0 pasan
por un punto jo.
3. Una circunferencia variable es tangente a una recta l en un punto jo O de l . Cules son los lugares
geomtricos de los puntos de contacto en el crculo generados por rectas tangentes paralelas a una
recta ja m?
4. Dados dos cuadrados de lados correspondientemente paralelos, qu sucede con las rectas que
unen vr ces correspondientes? podemos decir lo mismo de dos rectngulos de lados correspondientemente paralelos?
5. ABCD y A0 B 0 C 0 D0 son dos rectngulos tales que los lados ms largos de cada uno, AB y A0 B 0 , se
ubican sobre una misma recta l . Demuestre que los rectngulos son semejantes (el cociente entre
el largo y el ancho es igual) si y slo si las rectas BB 0 , CC 0 y l concurren.
6. Dadas dos tringulos rectngulos issceles no necesariamente iguales, demuestre que si sus hipotenusas se ubican sobre una misma recta, existe una homotecia que transforma a un tringulo en
el otro. D una manera de construir el centro de homotecia.
63

7. El problema de los bordes: Dado un polgono, se construye otro polgono en su interior obtenido
al bordear los lados con una banda de ancho constante a. El problema estudia lo siguiente: Las
rectas que unen vr ces correspondientes AA0 , BB 0 , CC 0 ; : : : son concurrentes?

En la gura anterior se muestra el caso cuando el polgono original es un pentgono. Demuestre


que
a) Si el polgono original es un tringulo, la respuesta es S.
b) Si el polgono original es un cuadrado, la respuesta tambin es S.
c) Si el polgono original es un rectngulo pero no es cuadrado, la respuesta es por lo general NO. Dentro de este caso, cundo la respuesta ser S? Generalice sus ideas para otros
cuadrilteros.
d) Si el polgono original es un rombo, la respuesta es s. En general, demuestre que si el polgono original es convexo y admite una circunferencia tangente a todos sus lados, entonces
la respuesta ser S.
e) Si existe una homotecia que transforma a un polgono en el otro la respuesta es S, y viceversa.
f) Qu sucede cuando el polgono original es regular?
8. Dado un tringulo 4ABC y un punto M sobre el lado BC , sean M1 la proyeccin ortogonal de M
sobre AB , M2 la proyeccin ortogonal de M1 sobre CA, y P 0 la proyeccin ortogonal de M2 sobre
BC . El obje vo del problema es determinar todos los puntos M (si existen) tales que el recorrido
regresa a M , ie, M 0 = M . Para ello:
1 Tome un punto cualquiera P sobre BC y construya P1 , P2 y P 0 .

2 Dena I como la interseccin de las rectas P P1 y P2 P 0 , y J como la interseccin de las rectas


AI con BC .
3 Verique que J es una solucin al problema (aplique una homotecia con centro en A y que
transforme a I en J ).
64

4 En el sen do lgico contrario: Suponga que M es un punto solucin del problema, demuestre
que a par r del recorrido que determina M es posible reconstruir el recorrido que determina
cualquier otro punto P (aplique una homotecia convenientemente).
9. Demuestre que toda homotecia con razn nega va puede ser escrita como la composicin de una
homotecia con razn posi va y una reexin.
10. Demuestre que HB; 1

 HA;k es una traslacin. Qu traslacin es?


Demuestre que toda homotecia HO;k es equivalente al producto de la homotecia HO ;k por una
!
k

11.

traslacin paralela a OO1 . Para ello, siga los siguientes pasos:

a) Dado un punto cualquiera A, construya A0 = HO;k (A).


b) Construya A00 = HO1 ;k (A) y O00 = HO1 ;k (O).

c) Demuestre que OA0 A00 O00 es un paralelogramo y concluya.

12. Dada una homotecia no trivial (razn de homotecia k 6= 1), demuestre que el centro de homotecia
es el nico punto jo y que las rectas que pasan por dicho centro son las nicas rectas jas tras la
homotecia.11
13. Demuestre que la homotecia es una transformacin directa, es decir, no cambia la orientacin de
los ngulos.
14. Demuestre que una homotecia con razn de homotecia k = 1 es la transformacin iden dad.
15. Demuestre que la transformacin inversa de la homotecia HO;k es otra homotecia, la cual est
denida por HO; 1 .
k

16. Demuestre que el conjunto de homotecias y traslaciones cumplen tener estructura de Grupo con
la composicin de funciones como operacin, es decir, forman un Grupo de Transformaciones.
17. Dado un punto arbitrario O, demuestre que el conjunto de homotecias concntricas en O forman
un Grupo de Transformaciones.12
18. Demuestre que el producto de tres homotecias es o bien una homotecia o una traslacin. Generalice el resultado para 4; 5; : : : homotecias.
19. Sea  la transformacin iden dad; se dice que una transformacin  es involutoria si  2 = .
Determine todas las homotecias que son involutorias.
20. Demuestre que, dada una homotecia H, el entero posi vo n ms pequeo para el cual Hn = 
a) es 1 si y slo si H = ,
b) es 2 si y slo si la razn de homotecia es

1 (reexin puntual), y

c) no existe en cualquier otro caso.


11

Punto o recta ja tras la homotecia quiere decir que al aplicar la homotecia a dicha gura, el resultado (o imagen) es la
misma gura.
12
Dos homotecias son concntricas si comparten el centro de homotecia.
65

21. Demuestre que la homotecia de una recta o un plano que pase por el centro de homotecia, es la
misma gura.
22. Demuestre que la homotecia de una circunferencia o una esfera con respecto a su centro, es una
circunferencia o una esfera concntrica.
23. U lizando las propiedades de las homotecias, demuestre el Teorema de Menelao:
a) Suponga que el 4ABC es cortado por la recta transversal O1 O2 O3 , tal que O1 est sobre AB ,
O2 sobre BC y O3 sobre CA. Calcule el valor de la razn de homotecia k1 tal que HO1 ;k1 (A) =
B ; calcule tambin el valor de la razn de homotecia k2 para que HO2 ;k2 (B ) = C .

b) Demuestre que HO2 ;k2 HO1 ;k1 es otra homotecia que transforma A en C ; determine adems
cul es centro y la razn de esta nueva homotecia.
c) Demuestre el Teorema de Menelao

AO1 BO2 CO3


= 1
O1 B O2 C O3 A
24. Demuestre el Teorema de Varignon u lizando propiedades de homotecias: Dado un cuadriltero
cualquiera ABCD, sean P Q, R, S , los puntos medios de AB , BC , CD , DA, respec vamente.
a) Construya una homotecia que transforme al segmento P Q en el segmento AC .
b) Construya una homotecia que transforme al segmento AC en el segmento SR.
c) Averige qu po de transformacin es la composicin de estas dos homotecias.
d) A par r de lo anterior, concluya que ABCD es un paralelogramo.
e) Deniendo M y N como los puntos medios de AC y BD, respec vamente, repita el proceso
anterior para demostrar que P MRN y QNSM son paralelogramos tambin.
f) Demuestre adems que P R, QS y MN concurren. Este punto es llamado el Centroide de
ABCD.
25. Dado un tringulo 4ABC sea M un punto variable sobre el lado AB . N es la interseccin de la AC
con la recta que pasa por M y es paralela a BC . P es la interseccin de BN con CM . Demuestre
que B describe una mediana del tringulo ABC cuando M vara.
26. Sean ABCD y AB 0 C 0 D0 dos paralelogramos con igual orientacin. Demuestre que BD

k B 0 D0 .

27. Sea ABCD un cuadriltero concclico convexo. Demuestre que las cuatro rectas trazadas con estas
siguientes caractersicas son concurrentes:
pasa por el punto medio de un lado del cuadriltero, y
es perpendicular al lado opuesto.
Pista: Llamando K a la interseccin de las cuatro rectas del problema, busque la relacin que existe
entre el circuncentro, el centroide y K .
66

28. Los vr ces B y C de un tringulo 4ABC estn jos, determine el lugar geomtrico que describen el centroide y los puntos medios de los lados AB y AC si A vara sobre una recta o una
circunferencia.
29. En un cuadriltero ABCD, los vr ces A, B y C estn jos, y el punto D se mueve de tal manera
que el lado AD siempre ene la misma medida. Determine el lugar geomtrico de
a) los puntos medios de AD, BD y CD,
b) el punto medio del segmento RS , que ene por extremos a los puntos medios de AC y BD,
respec vamente.
30. Resuelva el problema anterior, pero suponiendo que D se mueve de tal forma que el \ADB se
man ene constante.
31. Sean l y m dos rectas jas que se cortan en O; desde un punto P se construyen las proyecciones
A y B a l y m, respec vamente. Qu lugar geomtrico describe P si la recta AB vara man endo
una inclinacin constante.
32. Encuentre una homotecia dis nta de la iden dad, que transforme a un paralelogramo en s mismo.
33. Encuentre dos homotecias dis ntas de , que dejen invariantes a las bases de un trapecio.
34. Circunferencias tangentes, rectas tangentes y centros de homotecia:
a) Dada una circunferencia , si P es un punto cualquiera sobre y k 6= 1, demuestre que
HP;k ( ) es una circunferencia tangente a en P (si k > 0, las circunferencias son tangentes
internas, mientras que si k < 0 son circunferencias tangentes externas). Cul es el radio de
la circunferencia resultante?
b) Demuestre el recproco del resultado anterior: si dos circunferencias son tangentes en P (ya
sea circunferencias tangentes internas o externas), existe una homotecia de centro P y razn
k 6= 1 que transforma a una circunferencia en la otra. Cul es la razn de homotecia?
c) Demuestre que si las circunferencias son tangentes externas en P , el punto de interseccin
de las dos rectas tangentes comunes que no pasan por P es el otro centro de homotecia de
las circunferencias. Cul es la razn de homotecia?
d) Dadas dos circunferencias una fuera de la otra, demuestre que las dos rectas tangentes comunes externas concurren en un centro de homotecia de las circunferencias (a menos que
los radios sean iguales, en tal caso, estas tangentes quedan paralelas), y que las dos rectas
tangentes comunes internas concurren en el otro centro de homotecia de las circunferencias.
e) Dadas dos circunferencias de radios dis ntos y secantes en dos puntos dis ntos, demuestre
que las dos rectas tangentes comunes se cortan en un centro de homotecia de las circunferencias.
35. Denote por d (P; l ) la distancia de un punto P a una recta l . Sean l y m dos rectas jas y k una
constante.
a) Si l y m son rectas secantes en un punto O, demuestre que el lugar geomtrico de los puntos
P;l ) = k , son dos rectas que pasan por O .
P tales que dd((P;m
)
67

b) Si l k m, el lugar geomtrico descrito anteriormente, es una recta paralela a l y m, y se ubica


de tal forma que el cociente de las distancia a l y m es igual a la constante k .
c) Si en par cular k = 1, las rectas resultantes son la bisectriz interna y la bisectriz externa del
ngulo formado por l y m.
d) Cul es el lugar geomtrico de los centros de las circunferencias tangentes simultneamente
a l y m?
36. Centros de homotecia de tres circunferencias:
a) Dadas dos circunferencias no concntricas, demuestre que sus centros de homotecia dividen
armnicamente al segmento que une los centros de las circunferencias.
b) Dadas tres circunferencias cuyos centros forman un tringulo, demuestre que los seis centros de homotecia de tomar dos a dos a las circunferencias, forman cuatro ternas de puntos
alineados.
37. U lizando propiedades de tringulos homot cos, demuestre que las tres medianas de un tringulo concurren.
38. Recta de Euler: Demuestre u lizando homotecia que el ortocentro, el circuncentro y el centroide
de un mismo tringulo, estn alineados.
Pista: note que el tringulo medial es homot co al tringulo original.
39. Circunferencia de los 9 puntos: Dado un 4ABC , sea el circuncrculo y H el ortocentro. Demuestre que HH; 1 ( ) es la circunferencia de los 9 puntos del 4ABC . Qu propiedad ene el centro
2
de la circunferencia de los 9 puntos? Cules son los dos centros de homotecia de estas dos circunferencias?
Pista: Demuestre que la reexiones del ortocentro con respecto al los lados y con respecto a los
puntos medios de los lados, pertenecen a .
40. Una circunferencia variable es tangente a una recta ja l en un punto jo A. Determinar el lugar
geomtrico de los puntos de contacto de las rectas tangentes a la circunferencia con inclinacin
ja.
41. Una circunferencia variable es tangente a dos rectas secantes jas. Determinar el lugar geomtrico
de los puntos de contacto de las rectas tangentes a la circunferencia con inclinacin ja.
42. Dos circunferencias jas de radios dis ntos, son tangentes en A. Sea AB una cuerda variable sobre
una circunferencia, y AC una cuerda variable sobre la otra cirunferencia, tal que AB ? AC .
a) Demuestre que la recta variable BC pasa por un punto jo.
b) Determine el lugar geomtrico que describe el punto medio de BC .
c) Determine el lugar geomtrico que describe la proyeccin de A sobre BC .
43. Construya una circunferenia cumpliendo los requisitos de cada uno de los siguientes casos:

68

a) Dada una circunferencia , una recta l y un punto A que puede estar ubicado sobre o sobre
l , construya una circunferencia tangente a y tangente a l , de tal manera que A sea un punto
de tangencia.
b) Construya una circunferencia que pase por un punto dado y que sea tangente a dos rectas
secantes dadas.
c) Construya una circunferencia que sea tangente a una circunferencia dada y a dos rectas secantes dadas.
44. Construya el tringulo 4ABC en cada uno de los casos siguientes:
a) Conociendo el ngulo en A y las medianas respec vas a B y C .
b) Conociendo los ngulos B y C , y la distancia OH .
c) Conociendo el vr ce A, el centroide G y sabiendo que B y C pertenecen a dos rectas (o
circunferencias) dadas.
d) Conociendo la razn de los lados AB y AC , el ngulo A y el permetro del tringulo.
e) Conociendo los lados AB y AC y la bisectriz interna AD .
45. Un Cuadriltero Ortocntrico es la gura formada por los vr ces de un tringulo y su ortocentro.
a) Demuestre que en un cuadriltero ortocntrico, cada vr ce es el ortocentro del tringulo
formado por los tres vr ces restantes.
b) Demuestre que los cuatro centroides de los tringulos formados por los vr ces de un cuadriltero ortocntrico, forman otro cuadriltero ortocntrico homot co al original.
c) Demuestre que la circunferencia de los 9 puntos de un cuadriltero ortocntrico es concntrica a la circunferencia de los 9 puntos del cuadriltero formado por los cuatro centroides.

5.4.

Composicin de Homotecias

Teorema 5.3. Producto de dos Homotecias: Dadas dos homotecias HO1 ;k1 y HO2 ;k2 , si alguna de ellas es
la transformacin iden dad, el producto de stas es de nuevo una homotecia y dicho producto es adems
conmuta vo; en cualquier otro caso, se cumple que:
1. Independientemente de los valores que tome k1 o k2 , si O1 coincide con O2 , el producto HO2 ;k2 
HO1;k1 es otra homotecia HO;k1k2 cuya razn de homotecia es k1k2 y su centro de homotecia es
O = O1 = O2 . En este caso, las homotecias son llamadas homotecias concntricas, y el producto
es conmuta vo, es indiferente cul de las dos homotecias se realiza primero.
2. Si k1 6= 1, k2 =
6 1 y O1 no coincide con O2, el producto no es conmuta vo, y se subdivide en dos
casos:
a) Si k1 k2 6= 1, el producto HO2 ;k2  HO1 ;k1 es otra homotecia HO;k1 k2 cuya razn de homotecia
k2 1
1O
es k1 k2 , su centro de homotecia O se encuentra sobre la recta O1 O2 y cumple O
OO2 = k2 (k1 1) .

HO ;k  HO ;k
!
k )O O .

b) Si k1 k2 = 1, el producto
equivalentemente (1

69

!
es una traslacin cuyo vector es ( k1k1 1 )O1 O2 , o

Demostracin. Si alguna de las homotecias es la transformacin iden dad, el resultado es evidente. Por
otra parte, el siguiente argumento es vlido para el resto de casos, por comodidad se u lizar notacin
vectorial, pero la prueba sint ca es muy parecida y se deja como ejercicio:

Dado el vector AB , HO1 ;k1 (AB ) = A1 B1 con A1 B1 = k1 AB , y HO2 ;k2 (A1 B1 ) = A2 B2 con A2 B2 =
!
!
!
k2 A1 B1 , entonces A2 B2 = k1 k2 AB . Esto implica que HO2 ;k2  HO1 ;k1 es una homotecia de razn k1 k2 a
menos que k1 k2 = 1, en cuyo caso es una traslacin.
A con nuacin se completan las pruebas
1. Sin importar los valores que tome k1 o k2 , si O = O1 = O2 , la homotecia HO;k1 k2 ene el mismo
efecto neto que HO2 ;k2  HO1 ;k1 y que HO1 ;k1  HO2 ;k2 , esto completa la prueba.

2. En cambio, k1 =
6 1, k2 =6 1 y O1 no coincide con O2
a) Si k1 k2 6= 1, ya se demostr que el producto de las dos homotecias dadas es otra homotecia;
a con nuacin se demostrar que O, el centro de esta nueva homotecia, pertenece a la recta
O1 O2 y cumple la relacin OOO1 O2 = k2k(k2 1 11) ; de nuevo, se hace tanto la prueba sint ca como
la vectorial:
Primera demostracin: Se supone conocida la ubicacin de O, el centro de la homotecia producto. Sea O0 = HO1 ;k1 (O), dado que O es punto jo en la homotecia producto, debe cum!0 = k O !
!
plirse que O =(HO2 ;k2 (O0 ). Por la denicin
de
estas
homotecias,
O
O
O
y
O
O=
1
1
1
2
)
!
!
!
!
!
!
!
!
0
0
k2 O2 O = k2 O2 O + OO1 + O1 O , entonces O2 O = k2 O2 O + k2 OO1 + k2 k1 O1 O, de
donde (1

k2 )O2 O = k2 (k1

1)O1 O, por lo tanto

O1 O
k2 1
=
!
OO2 k2 (k1 1)
lo cual demuestra que O1 , O2 y O estn alineados (los vectores son ml plos escalares) y la
relacin buscada.
70

Segunda demostracin: La demostracin anterior puede realizarse tambin sint camente,


sin embargo, se mostrar una demostracin alterna va. Dado un punto A que no pertence
a la recta O1 O2 , sea A1 = HO1 ;k1 (A) y A2 = HO2 ;k2 (A1 ); la homotecia producto, que transforma A en A2 , cumple que su centro de homotecia est sobre la recta AA2 , llamemos OA a
la interseccin de AA2 con O1 O2 , por el teorema de Menelao aplicado al 4O1 A1 O2 con la
transversal AA2 OA se ene
(
)(
)(
)
O1 A
A1 A2
O2 OA
= 1
AA1
A2 O2
OA O1
O2 A 2
A1 A2
1
1A
pero OO11AA1 = k1 lo cual implica O
AA1 = k1 1 ; anlogamente, como O2 A1 = k2 entonces A2 O2 =
1 k2
k2 , y sus tuyendo

k1

)(

k2

k2

O2 OA
= 1
OA O1
O2 OA
(k1 1)k2
=
OA O1
1 k2
O1 OA
k2 1
=
OA O2
k2 (k1 1)

Si ahora se toma un segundo punto B que no pertenezca a O1 O2 y se repite el mismo procedimiento, denendo B2 = HO2 ;k2  HO1 ;k1 (B ) y OB como la interseccin de BB2 con O1 O2 ,
se puede demostrar que
O1 OB
k2 1
=
OB O2 k2 (k1 1)
por lo que OA = OB , y dado que el centro O de la homotecia producto se encuentra tanto
en la recta AA2 como en la recta BB2 , debe cumplirse que O = OA = OB y por tanto

O1 O
k2 1
=
OO2 k2 (k1 1)
La relacin buscada.
Ahora se discu r si el producto de dos homotecias no concntricas es conmuta vo o no lo
es en este caso. Note que si se invierte el orden del producto, los roles de k1 y k2 se invierten
71

y por tanto la homotecia resultante en este caso tendra como centro a un punto O tal que
O1 O = k1 1 ; para que la homotecia producto sea comnmuta va, O debera coincidir
O  O2
k1 (k2 1)
con O, pero eso equivale a

k2 1
k1 1
=
k2 (k1 1) k1 (k2 1)

Desarrollando esta expresin y simplicando, se llega a la condicin necesaria k1 k2 = 1, lo


cual de por s contradice la hiptesis, sin embargo, si todava insi ramos, al sus tuir esta
condicin en la relacin que probamos13

O1 O
k2 1
k 1
k 1
=
= 2
= 2
= 1
OO2 k2 (k1 1) k2 k1 k2 1 k2
Pero esto obliga a que O se aleje innitamente de O1 y O2 y dicho producto se deforma en
una traslacin, tal comos se dijo en un teorema anterior. En conclusin, si los dos centros de
homotecias no coinciden, k1 6= 1, k2 6= 1 y k1 k2 6= 1, el producto de estas homotecia no es
conmuta vo.
b) Si k1 k2 = 1, ya se demostr que el producto de las dos homotecia es una traslacin, resultado que se conrm nuevamente en el el prrafo anterior. A con nuacin se demostrar que
!
la traslacin ene por vector a ( k1k1 1 )O1 O2 .
Primera demostracin: Dado un punto A, sean A1 = HO1 ;k1 (A) y A2 = HO2 ;k2 (A1 ); si A
pertence a O1 O2 , tanto A1 como A2 tambin pertenecen a O1 O2 y el resultado se ob ene
rapidamente haciendo un poco de lgebra; por otra parte, si A no pertence a O1 O2 , por la
denicin de homotecia, O1 A1 = k1  O1 A y O2 A2 = k2  O2 A1 , pero dado que k1 k2 = 1 se
ene

O1 A1
1
OA
= k1 = = 2 1
O1 A
k2 O2 A2

U lizando el recproco del teorema de Thales aplicado al 4O1 A1 O2 se concluye que O1 O2


AA2 ; observe 4O1 A1 O2 ' 4AA1 A2 , y la razn de semejanza es

O1 O2 O1 A1
k1
=
=
AA2
AA1
k1 1
Esto signica que independientemente del punto A que se escoja, AA2 es un segmento paralelo a O1 O2 y su magnitud slo depende de k1 y O1 O2 , es decir, es constante; esto se resume
!
!
en la relacin vectorial AA2 = k1k1 1 O1 O2 ; por lo tanto, el producto de las homotecias es una
!
traslacin que ene al vector k1k1 1 O1 O2 como parmetro.
Observe que k2 =
6 1 porque en caso contrario k1 = 1 dado que k1 k2 = 1 y ambas homotecias seran la transformacin
iden dad.
13

72

73

Segunda demostracin: De nuevo, se toma un punto A y las imgenes A1 y A2 , tal como


!
! !
!
antes, sabiendo que O1 A1 = k1 O1 A, O2 A2 = k2 O2 A1 y k2 = k11 .

AA2 = AA
+A A
( 1 ! 1 2!) ( !
!)
= O1 A1 O1 A + A1 O2 + O2 A2

=
=
=
=

!
OA
1

k1

k1

( !
)
!
1 !
O A + A1 O2 + k2 O2 A1
1
k1 1 1
)
(
)
!
!
1
1
O1 A1 + 1
A1 O2
k1
k1
)(
!
!)
1
O1 A1 + A1 O2
k1
)
!
1
O1 O2

La relacin buscada.
Finalmente, si se invierte el orden del producto de las homotecias, haciendo un razonamiento
!
anlogo se llegara a una traslacin con vector k2k2 1 O2 O1 , para que el producto sea conmuta vo, debera cumplirse que esta l ma traslacin coincide con la anterior, pero eso slo se
dar si

k1

k1

k2

k2

Lo cual, dado que k1 k2 = 1 es equivalente a (k1 1)2 = 0, es decir, k1 = 1, obligando


tambin a k2 = 1, por lo que se cae en el caso trivial cuando las dos homotecias son transformaciones iden dad. En conclusin, si O1 no coincide con O2 , k1 6= 1, k2 6= 1 y k1 k2 = 1,
el producto tampoco es conmuta vo.

Teorema 5.4. El producto en cualquier orden de una homotecia con una traslacin, es otra homotecia
con la misma razn.

Demostracin. Sean HO;k y T!


 la homotecia y la traslacin que se u lizarn en la composicin; sea AB
un vector cualquiera,
( !)
( ( !))
HO;k  T AB
= HO;k T AB
( !)
= HO;k AB

= k AB

74

Dado que la traslacin deja invariante al vector AB y la homotecia los mul plica por un escalar; de aqu se
concluye que la transformacin producto debe ser una homotecia con razn k , y el centro de homotecia
se encuentra en la interseccin de las rectas AA0 y BB 0 , donde A0 = HO;k T (A) y B 0 = HO;k T (B ).
Si se intercambia el orden y se realiza la composicin T  HO;k , anlogamente se demuestra que esta
transformacin debe ser otra homotecia de razn k y se deja como ejercicio.

75

Anda mungkin juga menyukai